Sunteți pe pagina 1din 50

1.

With reference to the Prime government subsidy through designated


Minister's Employment Generation banks for eventual disbursal to the
Programme (PMEGP), consider beneficiaries / entrepreneurs directly into
the following statements: their bank accounts,
(1) PMEGP is a credit-linked Statement 3 is correct.
subsidy programme aimed at Any individual above 18 years of age is
generating self-employment eligible. For setting up of projects costing
opportunities through above Rs. 10 lakhs in the manufacturing
establishment of micro- sector and above Rs. 5 lakhs in the
enterprises in the non-farm business /service sector, the
sector by helping traditional beneficiaries should possess at least VIII
artisans and unemployed standard pass educational qualification.
youth. Benefit can be availed under PMEGP for
(2) The Ministry of Finance is the setting up of new units only.
nodal agency for the Source:
implementation of PMEGP. http://pib.nic.in/newsite/mbErel.aspx?reli
(3) Any individual above 18 d=169467
years of age and 8th https://msme.gov.in/schemes/pm-
standard pass educational employment-generation-program-and-
qualification is eligible for other-credit-support-schemes
PMEGP.
Which of the statements given 2. Which of the following is the
above are correct? objective of ‘SAMARTH’ scheme?
(a) 1 and 2 only (a) Strengthening of the Self
(b) 1 and 3 only Help Groups
(c) 2 and 3 only (b) Capacity building in Textile
(d) 1, 2 and 3 Sector
(c) Physical training of young
Answers: (b) women
Explanation: (d) Self-employment for rural
Statement 1 is correct. youth
PMEGP is a credit-linked subsidy
programme aimed at generating self- Answers: (b)
employment opportunities through Explanation:
establishment of micro-enterprises in the SAMARTH - Scheme for Capacity
non-farm sector by helping traditional building in Textile Sector, got launched
artisans and unemployed youth. by the Ministry of Textiles for the training
Statement 2 is incorrect. and employability of the youth in the
The scheme is implemented by Khadi booming Textile Sector of India. It is part
and Village Industries Commission of the Skill India Mission. It aims to
(KVIC) functioning as the nodal agency provide skill development training to
at the national level. At the state level, around 10 lakh youth (9 lakh in the
the scheme is implemented through organised and 1 lakh in the traditional
State KVIC Directorates, State Khadi and Textile Sector) over a period of 3 years
Village Industries Boards (KVIBs), (2017-2020) with an outlay of 1300 crore
District Industries Centres (DICs) and rupees, so as to achieve the
banks. In such cases KVIC routes

Prelim IAS Test Series (2019) – GS Test 10 (10.02.2019)


Economy 2 and Current Affairs Nov 2018 1
government's vision of increasing India’s (NBFC) for mobilising extra-budgetary
textile export to 300 billion USD by 2025. resources for building crucial
Source: infrastructure in the higher educational
http://pib.nic.in/newsite/PrintRelease.asp institutions under Central Government. In
x?relid=179269 the existing arrangement, the entire
principal portion is repaid by the
3. With reference to Revitalising institution over ten years, and the interest
Infrastructure and Systems in portion is serviced by the Government by
Education (RISE) scheme, providing additional grants to the
consider the following statements: institution.
(1) The scheme will focus only Source:
on infrastructure http://pib.nic.in/newsite/PrintRelease.asp
development in premier x?relid=180371
higher educational http://www.pib.nic.in/Pressreleaseshare.a
institutions. spx?PRID=1526148
(2) The scheme will be financed
by the Higher Education 4. National Pension Scheme (NPS) is
Financing Agency (HEFA) an attempt by the government to
which is a Non-Profit, Non- create a pensioned society in India.
Banking Financing Company With reference to National Pension
(NBFC). Scheme (NPS), consider the
Which of the statements given following statements:
above is/are correct? (1) It is available to government
(a) 1 only employees only.
(b) 2 only (2) NPS is regulated by the
(c) Both 1 and 2 Pension Fund Regulatory
(d) Neither 1 nor 2 and Development Authority
(PFRDA).
Answers: (c) Which of the statements given
Explanation: above is/are correct?
Statement 1 is correct. (a) 1 only
In the Budget 2018-19 announcement (b) 2 only
has been made to launch a major (c) Both 1 and 2
initiative named ‘‘Revitalising (d) Neither 1 nor 2
Infrastructure and Systems in Education
(RISE)’’ with a total investment of Rs. Answers: (b)
1,00,000 crore in next four years by Explanation:
structuring Higher Education Financing Statement 1 is incorrect.
Agency (HEFA). The objective of the Any citizen of India, whether government
RISE is to step up investments in employee or non-government employee;
research and related infrastructure in whether resident or non-resident can join
premier educational institutions, including NPS.
health institutions by 2022. Statement 2 is correct.
Statement 2 is correct. National Pension System (NPS) is a
HEFA has been set up on 31st May 2017 voluntary defined contribution pension
by the Central Government as a Non- system administered and regulated by
Profit, Non-Banking Financing Company

Prelim IAS Test Series (2019) – GS Test 10 (10.02.2019)


Economy 2 and Current Affairs Nov 2018 2
the Pension Fund Regulatory and the ratio should stay relatively consistent.
Development Authority. Consequently, as the gross domestic
Value addition: product (GDP) grows, tax revenue
Conditions to join NPS should increase as well. However, in
 The subscriber should be between 18 cases of significant shifts in tax law or
and 65 years old as of the date of during severe economic downturns, the
submission of his/her application to ratio can shift, sometimes dramatically.
the Point of Presence (POP) / Point Economic downturns will result in lower
of Presence – Service Provider – Tax-to-GDP ratio. During these times,
Authorized branches of POP for NPS unemployment usually rises, and
(POP-SP). consumer spending decreases. As a
 The subscribers should comply with result, there are fewer property and
the Know Your Customer (KYC) consumption taxes collected. During
norms as detailed in the subscriber downturns, reduced consumption
registration form. significantly and quickly affects tax
 Should not be Un-discharged receipts, pushing the Tax-to-GDP ratio
insolvent and individual of unsound downward.
mind. Source:
Source: 1. VAJIRAM AND RAVI Current
http://pib.nic.in/newsite/PrintRelease.a Affairs for October 2018 Page 57
spx?relid=177957 2. https://www.investopedia.com/ter
ms/t/tax-to-gdp-ratio.asp
5. Which of the following are possible
implications of a declining Tax to 6. Consider the following statements
GDP ratio? regarding 'Global Skills Park':
(1) Lower rate of growth (1) It will be set up at Indore in
(2) Lower consumer spending India, aided by the Asian
(3) Ascending unemployment Development Bank (ADB).
(4) Fewer tax collections (2) It will be the country's first
Select the correct answer using the multi-skills park.
code given below: (3) It will have training facilities
(a) 1 and 2 only focusing on skills for
(b) 2 and 3 only manufacturing, service and
(c) 1, 3 and 4 only advanced agricultural jobs.
(d) 1, 2, 3 and 4 Which of the statements given
above are correct?
Answer: (d) (a) 1 and 2 only
Explanation: (b) 1 and 3 only
The tax-to-GDP ratio is the ratio of a (c) 2 and 3 only
nation's tax revenue relative to its gross (d) 1, 2 and 3
domestic product (GDP). Some countries
aim to increase the Tax-to-GDP ratio to Answer: (c)
address deficiencies in their budgets. Explanation:
Policymakers and analysts use the Tax- Statement 1 is incorrect and
to-GDP ratio to compare tax receipts statement 2 is correct:
from year to year. In most cases, as the
taxes are related to economic activity,

Prelim IAS Test Series (2019) – GS Test 10 (10.02.2019)


Economy 2 and Current Affairs Nov 2018 3
India and Asian Development Bank Indian Companies Act (2013) under
(ADB) have signed a $150 million loan Section 135 mandates that Indian
agreement to corporates, public and private, must
establish the country's first multi-skills allocate at least 2 per cent of their net
park in Bhopal to create a more skilled profits for CSR (Corporate Social
workforce. Responsibility).
Statement 3 is correct. Option (b) is incorrect.
It will have training facilities focusing on The Ministry of Corporate Affairs (MCA)
skills for manufacturing, service and data shows that the bulk of the CSR
advanced agricultural jobs, benefitting money (almost 75 per cent) is allocated
about 20,000 trainees and trainers. to just three sectors i.e. education, health
Value addition: (including sanitation and water) and rural
It will engage international Technical and poverty.
Vocational Education and Training Option (c) is correct.
(TVET) partners to support advanced The MCA data also reveals a skew in the
training who will bring global best distribution of the CSR funds. Almost 40
practices in TVET management, training per cent of the money goes to just a few
infrastructure, industry cooperation and relatively well-developed states such as
quality assurance. Maharashtra, Gujarat, Karnataka, Tamil
Source: Nadu, Andhra Pradesh and Telangana.
VAJIRAM AND RAVI Current Affairs Thus, this model in short aggravates
for October 2018 Page 61 rather than alleviates existing regional
and social disparities.
7. Which of the following statements Option (d) is incorrect.
regarding Corporate Social No such body exists right now. Many
Responsibility (CSR) is correct? challenges have raised the need for a
(a) The Indian Companies Act, different model for CSR expenditure such
2013 mandates that Indian as corporates pool their CSR funds into a
corporates, public and common CSR trust and allow an
private, must allocate at least autonomous body to manage and
5 per cent of their net profits disburse the funds. This body should be
for CSR. a confederation of corporates, NGOs,
(b) The CSR funds are equally domain experts and government.
distributed across all sectors. Source: VAJIRAM AND RAVI Current
(c) 40 per cent of the money Affairs for September 2018 Page 73
goes to just a few relatively
well-developed states such 8. With respect to the National Digital
as Maharashtra, Gujarat, Communications Policy 2018
Karnataka etc. (NDCP), consider the following
(d) A common CSR trust which is statements:
an autonomous body (1) It envisages four million
manages and disburse the additional jobs and boost
funds collected under CSR. sectoral contribution to 8% of
GDP
Answer: (c) (2) It will ensure universal
Explanation: broadband connectivity and
Option (a) is incorrect.

Prelim IAS Test Series (2019) – GS Test 10 (10.02.2019)


Economy 2 and Current Affairs Nov 2018 4
ensure connectivity to all Source: VAJIRAM AND RAVI Current
uncovered areas. Affairs for October 2018 Page 65
(3) It also calls for establishment
of a National Digital Grid by 9. NPA is a loan or an advance where
creating a National Fibre principal/ interest payment remains
Authority to facilitate overdue for a period of more
development of open access than __ days in respect of a term
NextGen networks. loan.
Which of the statements given (a) 30
above is/are correct? (b) 60
(a) 1 only (c) 90
(b) 2 only (d) 12
(c) 2 and 3 only
(d) 1, 2 and 3 Answer: (c)
Explanation:
Answer: (d) NPA:
Explanation: Definition: A non-performing asset (NPA)
The Union Cabinet approved the new is a loan or advance for which the
Telecom Policy i.e. The National Digital principal or interest payment remains
Communications Policy 2018. overdue for a period of 90 days.
Statement 1 is correct: Description: Banks are required to
It aims to create four million additional classify NPAs further into Substandard,
jobs, draw $100 billion of investments Doubtful and Loss assets.
into the telecom industry by 2022, boost 1. Substandard assets: Assets which
sectoral contribution to 8% of GDP from have remained NPA for a period less
6% in 2017, besides backing the than or equal to 12 months.
principles of net neutrality. 2. Doubtful assets: An asset would be
Statement 2 is correct: classified as doubtful if it has remained in
Universal broadband: Under NDCP, the the substandard category for a period of
government plans to ensure universal 12 months.
broadband 3. Loss assets: As per RBI, “Loss asset
connectivity at 50 Mbps to every citizen, is considered uncollectible and of such
provide 1 Gbps connectivity to all gram little value that its continuance as a
panchayats by 2020 and 10 Gbps by bankable asset is not warranted,
2022 and ensure connectivity to all although there may be some salvage or
uncovered areas recovery value.”
Statement 3 is correct: Source:
It calls for establishment of a National 1. VAJIRAM AND RAVI Current
Digital Grid by creating a National Fibre Affairs For September 2018 Page
Authority to facilitate development of 70
open access next generation networks 2. https://economictimes.indiatimes.c
and creating a collaborative mechanism om/definition/non-performing-
between the Centre, states and local assets
bodies for common Rights of Way
(RoW), standardization of costs and 10. Consider the following statements
timelines about SAMPADA:

Prelim IAS Test Series (2019) – GS Test 10 (10.02.2019)


Economy 2 and Current Affairs Nov 2018 5
(1) It is an umbrella scheme agricultural produce, increasing the
encompassing all the ongoing processing level and enhancing the
schemes of the Ministry of Food export of the processed foods.
Processing Industries like Mega The following schemes will be
Food Parks, Integrated Cold implemented under PM Kisan SAMPADA
Chain etc. and not any new Yojana:
scheme.  Mega Food Parks
(2) This scheme is public sector  Integrated Cold Chain and Value
driven and provides more Addition Infrastructure
freedom to the entrepreneurs to  Creation/ Expansion of Food
choose the project location. Processing/ Preservation Capacities
(3) The scheme aims to enhance (Unit Scheme)
rural employment.  Infrastructure for Agro-processing
Which of the statements given Clusters
above is/are correct?  Creation of Backward and Forward
(a) 1 only Linkages
(b) 3 only  Food Safety and Quality Assurance
(c) 1 and 2 only Infrastructure
(d) 1, 2 and 3  Human Resources and Institutions
Source:
Answer: (b) http://mofpi.nic.in/Schemes/pradhan-
Explanation: mantri-kisan-sampada-yojana
Statement 1 is incorrect.
It also contains new schemes related to 11. India is known to be one of the
backward and forward linkages, agro- largest consumers of gold in the
processing clusters and increasing food world. Off late, we have seen a
processing and preservation capacities. slight dip in the demand of gold.
Statement 2 is incorrect. Which of the following reasons can
It is a private sector driven project be attributable to such a trend?
whereby public sector only plays the role (1) Prevention of Money
of an anchor. Laundering Act,
Statement 3 is correct. Demonetization, Mandatory
The scheme aims to enhance rural hallmarking to ensure purity
employment. standards
Additional Info: (2) India has also made it
The PM Kisan SAMPADA Yojana is a mandatory for customers to
comprehensive package which will result disclose their tax code, or
in creation of modern infrastructure with Permanent Account Number
efficient supply chain management from (PAN), for high-value gold
farm gate to retail outlet. It will not only purchases.
provide a big boost to the growth of food (3) High depreciation of the
processing sector in the country but also rupee on an average in
help in providing better returns to farmers 2018-2019.
and is a big step towards doubling of (4) Buyers have also sought
farmers income, creating huge alternative investments like
employment opportunities especially in the equity market off late.
the rural areas, reducing wastage of

Prelim IAS Test Series (2019) – GS Test 10 (10.02.2019)


Economy 2 and Current Affairs Nov 2018 6
Select the correct answer using the commerce platform should be provided
code given below: services in a “fair and non-discriminatory
(a) 1 only manner”. The marketplaces will not be
(b) 1 and 3 only allowed to offer deep discounts through
(c) 1, 2 and 3 only their in-house companies listed as
(d) 1, 2, 3 and 4 sellers.
Statement 3 is correct.
Answer: (d) E-commerce companies follow both
Explanation: marketplace model and inventory model.
Marketplace model- E-commerce
12. Consider the following statements Company provides an IT platform on a
with respect to the e-commerce digital or electronic network to act as
sector: facilitator between buyers and sellers
(1) In India, e-commerce without warehousing the products.
companies cannot sell Inventory model- Products are owned
products through companies, by the online shopping company. The
and of companies, in which whole process end-to-end, starting with
they hold equity stake. product purchase, warehousing and
(2) The e-marketplaces in India ending with product dispatch, is taken
can offer deep discounts care of by the company.
through their in-house Source:
companies listed as sellers. https://www.theweek.in/news/biz-
(3) The e-commerce companies tech/2018/12/27/ecommerce-rules-
follow both marketplace etailers-vendors.html
model and inventory model. https://www.investindia.gov.in/faqs/w
Which of the statements given hat-marketplace-and-inventory-based-
above is /are correct? model
(a) 1 and 2 only
(b) 1 and 3 only 13. ‘States' Start-up Ranking 2018’ is
(c) 3 only released by which of the following?
(d) 1, 2 and 3 only (a) Small Industries
Development Bank of India
Answer: (b) (SIDBI).
Explanation: (b) Department of Industrial
Statement 1 is correct. Policy and Promotion (DIPP).
Recently, government introduced (c) Ministry of Corporate Affairs.
changes in e-commerce norms: From (d) Ministry of Micro, Small and
February 1, 2019, e-commerce Medium Enterprises.
companies running marketplace
platforms - such as Amazon and Flipkart Answer: (b)
- cannot sell products through Explanation:
companies, and of companies, in which Option (b) is correct.
they hold equity stake. The Department of Industrial Policy and
Statement 2 is incorrect. Promotion (DIPP) announced results of
No seller can be forced to sell its the first ever States’ Start-up Ranking
products exclusively on any marketplace 2018. Gujarat ranked the best performer
platform, and that all vendors on the e- and Karnataka, Kerala, Odisha, and

Prelim IAS Test Series (2019) – GS Test 10 (10.02.2019)


Economy 2 and Current Affairs Nov 2018 7
Rajasthan are the top performers. States Maximum tenure in case of swapping
have been identified as leaders across between two linkage holders shall be
various categories such as Start-up lesser of the remaining tenure of the
policy leaders, incubation hubs, seeding contracts of the willing participants.
innovation, scaling innovation, regulatory This will be facilitated via an electronic
change champions, procurement platform where participants can register,
leaders, communication champions, and on a pre-decided time applicant can
North-Eastern leader, and hill state swap coal supplies with the registered
leader. participants.
Source: Statement 2 is correct.
http://pib.nic.in/newsite/PrintRelease.a Coal India would be the nodal agency for
spx?relid=186652 the swapping arrangement.
Source:
14. With reference to the 'Coal https://indianexpress.com/article/busi
Swapping Scheme', consider the ness/economy/coal-swapping-
following statements: scheme-extended-pvt-power-
(1) It is the coal supply producers-cement-steel-sectors-
rationalization arrangement 5505573/
available only to the state-run
power plants. 15. The term ‘Universal Basic Income’
(2) Coal India would be the (UBI) is often seen in the news. In
nodal agency for the this context which of the following
swapping arrangement. are potential benefits of Universal
Which of the statements given Basic Income?
above is/are correct? (1) Better targeting of poor
(a) 1 only (2) Promotion of liberty and
(b) 2 only freedom of choice.
(c) Both 1 and 2 (3) Administrative efficiency
(d) Neither 1 nor 2 Select the correct answer using the
code given below:
Answer: (b) (a) 1 and 3 only
Explanation: (b) 1 and 2 only
Coal Swapping scheme is a coal supply (c) 2 and 3 only
rationalization arrangement between two (d) 1, 2 and 3
willing consumers for swapping “full or
part of their entitled quantity” for a Answers: (d)
“minimum tenure of Explanation:
swapping/rationalisation of six months”. Universal Basic Income is a radical and
Statement 1 is incorrect. compelling paradigm shift in thinking
Recently, after allowing coal swapping about both social justice and a productive
among state-run power plants, the inter- economy.
ministerial task force (IMTF) has decided Statement 1 is correct. As there will be
to extend the supply rationalisation zero exclusion, the targeting of poor will
scheme to private power producers improve.
and non-regulated cement and steel Statement 2 is correct. The
sectors who are importing coal or have circumstances that keep individuals
domestic supply linkages. trapped in poverty are varied; the risks

Prelim IAS Test Series (2019) – GS Test 10 (10.02.2019)


Economy 2 and Current Affairs Nov 2018 8
they face and the shocks they face also Although, US has defined the shell
vary. The State is not in the best position companies under their Securities Act, as
to determine which risks should be a Shell Company is a company, other
mitigated and how priorities are to be set. than an asset-backed issuer, with no or
UBI liberates citizens from paternalistic nominal operations; and either:
and client relationships with the State. By  no or nominal assets
taking the individual and not the  assets consisting of cash and
household as the unit of beneficiary, UBI cash equivalents; or
can also enhance agency, especially of  assets consisting of any amount
women within the households. of cash and cash equivalents and
Statement 3 is correct. UBI can nominal other assets.
overcome weakness of existing welfare Statement 1 is correct.
schemes which are riddled with Theoretically, shell companies are
misallocation, leakages and exclusion of companies without active business
the poor. When the trinity of Jan-Dhan, operations or significant assets.
Aadhaar and Mobile (JAM) is fully Statement 2 is correct.
adopted the time would be ripe for a Not all shell companies are vehicles of
mode of delivery that is administratively illegal activities. There are many shell
more efficient. companies that work within legal limits
Source: (1) Economic Survey 2016-17, and do not have financial irregularities.
chapter 09 For example, a company may separate
https://www.indiabudget.gov.in/es201 its HR function into another company
6-17/echap09.pdf altogether or some companies could
have been started to promote start-ups
16. Consider the following statements by raising funds.
with respect to shell companies: Source:
(1) They are companies without https://www.livemint.com/Money/gFqq
active business operations or UTa6pYGWVkjyHh58QO/What-are-
significant assets. shell-companies.html and
(2) Not all shell companies are https://www.moneycontrol.com/news/
indulged in illegal activities. business/what-are-shell-companies-
Which of the statements given all-you-need-to-know-2361177.html
above is/are correct?
(a) 1 only 17. The 12th Five Year Plan had set
(b) 2 only 4% agriculture growth rate as a
(c) Both 1 and 2 target. In this context consider the
(d) Neither 1 nor 2 following statements:
(1) The growth rates of
Answer: (c) agriculture and allied sectors
Explanation: stagnated gradually during
In India shell companies are not defined this period (2012-2017).
under Companies Act 2013 or any other (2) The share of livestock in
legislation. However, some laws can help agriculture GVA (Gross
curbing illegal activities such as money Value Addition) gradually
laundering and can be used to target declined whereas the share
shell companies – Benami Transaction of crops increased.
(Prohibition) Amendment Act 2016, etc.

Prelim IAS Test Series (2019) – GS Test 10 (10.02.2019)


Economy 2 and Current Affairs Nov 2018 9
Which of the statements given 18. Consider the following statements
above is/are correct? regarding Minimum Support Price
(a) 1 only (MSP):
(b) 2 only (1) MSPs serve as a floor price
(c) Both 1 and 2 that is fixed by the
(d) Neither 1 nor 2 government as a long-term
guarantee for investment
Answer: (d) decisions of producers.
Explanation: (2) There is a significant regional
Statement 1 is incorrect: The growth discrimination in the
rates of agriculture & allied sectors procurement of food grains at
fluctuated at 1.5% in 2012-13, 5.6% in MSP in India.
2013-14, (-)0.2% in 2014-15, 0.7% in (3) MSPs are responsible for
2015-16 and 4.9% in 2016-17. The distorted cropping pattern in
uncertainties in growth in agriculture can India.
be explained by the fact that more than Which of the statements given
50 percent of agriculture in India is above are correct?
rainfall dependent which aggravates the (a) 1 and 2 only
production risks. (b) 1 and 3 only
(c) 2 and 3 only
(d) 1, 2 and 3

Statement 2 is incorrect: The Answer: (d)


Explanation:
agriculture sector witnessed a gradual
structural change in the 12th FYP period. Minimum Support Price (MSP) is a form
The share of livestock in GVA in of market intervention by the
Government of India to insure agricultural
agriculture increased gradually, the share
of the crop sector in GVA declined from producers against any sharp fall in farm
produce prices.
65 per cent in 2011-12 to 60 per cent in
Statement 1 is correct.
2015-16.
The minimum support prices are
announced by the Government of India
at the beginning of the sowing season for
certain crops on the basis of the
recommendations of the Commission for
Agricultural Costs and Prices (CACP).
Thus, MSPs served as a floor price that
was fixed by the government as a long-
term guarantee for investment decisions
of producers.
Statement 2 is correct.
The government procurement at MSP is
highly skewed in favour of some northern
States whereas highly discriminated
Source: Economic Survey 2017-18 against eastern States where it is
Volume 2 – Chapter 7 minimal or non-existent.
Statement 3 is correct.

Prelim IAS Test Series (2019) – GS Test 10 (10.02.2019)


Economy 2 and Current Affairs Nov 2018 10
MSP has put excessive focus on some Statement 1 is incorrect.
crops such as wheat, paddy and EPC (Engineering, procurement and
sugarcane at the expense of pulses, construction) model: Under this,
oilseeds, etc. Therefore, cropping government pays private players for the
patterns in several parts of the country project. The private player has no role in
are distorted. the project’s ownership, toll collection or
Additional Info: maintenance.
CACP considers several factors while Statement 2 is correct.
recommending the MSP for a commodity BOT (Build, Operate, Transfer) model:
such as cost of production, changes in Under this, the private player arranged all
input prices, international price situation, the finances for the project, while
inter-crop price parity, etc. Government collecting toll revenue or annuity fee from
announces minimum support prices the Government, as agreed. Private
(MSPs) for 24 mandated crops and Fair developers have an active role - they
and Remunerative Price (FRP) for build, operate and maintain the project
sugarcane. for a specified number of years before
Source: transferring the asset back to the
http://vikaspedia.in/agriculture/market government. In BOT-annuity, the toll
-information/minimum-support-price revenue risk is taken by the government,
while the private player is paid a pre-
19. Which of the following statements fixed annuity for construction and
is/are correct regarding different maintenance of project.
models of investments in Statement 3 is correct.
infrastructure? HAM (Hybrid-Annuity Model): It is a
(1) Under EPC (engineering, mix of the EPC (40%) and BOT (60%)
procurement, construction) models. In the first tranche,
model, private players have government/public authority releases 40
an active role in operation per cent of the total project cost in five
and maintenance of the tranches linked to milestones. The
project. balance 60 per cent is arranged by the
(2) Under BOT (Build, Operate, developer. It is a good trade-off,
Transfer) model, private spreading the risk between developers
players must arrange all the and the Government. Here, the
finances for the project. government pitches in to finance 40 per
(3) Under HAM (Hybrid-Annuity cent of the project cost - a sort of
Model) model, government viability-gap funding. This helps cut the
invests part funding of the overall debt and improves project
project. returns.
Select the correct answer using the Value Addition: Recently, Government
code given below: of India has approved Toll Operate and
(a) 1 only Transfer (TOT) model under which the
(b) 1 and 3 only public funded projects operational for two
(c) 2 and 3 only years shall be put to bid wherein the right
(d) 1, 2 and 3 of collection and appropriation of fee
shall be assigned for a pre-determined
Answer: (c) concession period (30 years) to
Explanation: Concessionaires (Developers/Investors)

Prelim IAS Test Series (2019) – GS Test 10 (10.02.2019)


Economy 2 and Current Affairs Nov 2018 11
against upfront payment of a lump sum (1) Economy is reeling under
amount to NHAI. hyperinflation
or deflation
Source: Vajiram and Ravi Indian (2) As an anti-terrorism measure
Economy Part B- Chapter 14, page no. (3) To tackle economic instability
214 (4) Transition to a new legal
tender
20. Consider the following statements Which of the statements given
regarding Global Foreign above is/are correct?
Exchange Committee: (a) 1 only
(1) It is a forum of Central (b) 4 only
Bankers and experts working (c) 2, 3 and 4 only
towards promotion of a robust (d) 1, 2, 3 and 4
and transparent forex market.
(2) The Committee is established Answer: (c)
under the aegis of the World Explanation:
Bank. Demonetisation can be undertaken under
(3) India is one of the members hyperinflationary conditions but not under
of the Committee. deflation.
Which of the statements given
above is/are correct? 22. Consider the following statements
(a) 1 only regarding “The Financial Action
(b) 1 and 2 only Task Force (FATF)”:
(c) 1, 2 and 3 (1) The mandate of the FATF is
(d) None of the above to counter money laundering,
terrorist financing and related
Answer: (a) threats to the integrity of
Explanation: international financial system.
Statement 1 is correct. (2) The FATF currently
Global Foreign Exchange Committee is a comprises 36 member
forum of Central Bankers and experts jurisdictions.
working towards promotion of a robust (3) Currently India is an observer
and transparent forex market. in the FATF, and will get its
Statement 2 is incorrect. membership in June, 2019.
It is under the aegis of the Bank of (4) FATF releases Black, Grey
International Settlements (BIS). It and White lists to maintain a
comprises of public and private sector record of countries that have
representatives from Forex committees a poor record in action
of 16 international Forex trading centres. against money laundering,
Statement 3 is incorrect. terrorist financing and other
India is soon going to be a part of it. such crimes.
Currently India is not its member. Which of the statements given
Source: https://www.globalfxc.org/ above are correct?
(a) 1 and 2 only
21. Under which of the following (b) 1, 2 and 3 only
conditions, policy makers might (c) 1, 2 and 4 only
resort to demonetization? (d) 1, 2, 3 and 4

Prelim IAS Test Series (2019) – GS Test 10 (10.02.2019)


Economy 2 and Current Affairs Nov 2018 12
Answer: (a) Answer: (d)
Explanation: Explanation:
Statement 1 is correct. Statement 1 is incorrect.
The objectives of the FATF are to set The Ministry of Food Processing
standards and promote effective Industries has launched the scheme.
implementation of legal, regulatory and NAFED will be the Nodal Agency to
operational measures for combating implement price stabilisation measures.
money laundering, terrorist financing and Statement 2 is incorrect.
other related threats to the integrity of the In the budget speech of Union Budget
international financial system. 2018-19, a new Scheme “Operation
Statement 2 is correct. Greens” was announced with an outlay
As of now, the FATF currently comprises of Rs. 500 crore to promote Farmer
36 member jurisdictions and 2 regional Producers Organizations (FPOs), agri-
organisations, the EU and the Gulf Co- logistics, processing facilities and
operation Council, collectively professional management.
representing most major financial centres Objectives:
in all parts of the globe.  Enhancing value realisation of
Statement 3 is incorrect. Tomato, Onion and Potato (TOP)
India became a member of the Financial farmers by targeted interventions to
Action Task Force (FATF) in 2010. The strengthen TOP production clusters
membership of FATF came nearly four and their FPOs, and
years after the country became an linking/connecting them with the
observer in 2006. market.
Statement 4 is incorrect.  Price stabilisation for producers and
FATF maintains two different lists of consumers by proper production
countries: Grey list & Black List planning in the TOP clusters and
introduction of dual use varieties.
23. In the Union Budget 2018-19, a  Reduction in post-harvest losses by
new Scheme “Operation Greens” creation of farm gate infrastructure,
was announced on the line of development of suitable agro-
“Operation Flood”. In this context, logistics, creation of appropriate
consider the following statements: storage capacity linking consumption
(1) The Ministry of Agriculture centres.
and Farmers' Welfare has  Increase in food processing
launched the scheme. capacities and value addition in TOP
(2) The aim of the scheme is to value chain with firm linkages with
promote horticulture and production clusters.
animal husbandry as an  Setting up of a market intelligence
alternative source of income network to collect and collate real
for the farmers. time data on demand and supply and
Which of the statements given price of TOP crops.
above is/are correct? Source:
(a) 1 only http://mofpi.nic.in/Schemes/operation-
(b) 2 only greens
(c) Both 1 and 2
(d) Neither 1 nor 2 24. The NSSO measures the
unemployment rate in India by

Prelim IAS Test Series (2019) – GS Test 10 (10.02.2019)


Economy 2 and Current Affairs Nov 2018 13
different methods. Consider the be first determined whether he/she
following statements in this context: belongs to the ‘Labour Force’ or not,
(1) The NSSO measures which in turn depends on the Activity
unemployment rate in the Status of the individual during the
country every 5 years. chosen reference period.
(2) Under the Usual Status Activity Status refers to the activity
method, the reference period situation in which the individual is found
is one year while under during the reference period with respect
Current Daily Status method to his participation in economic or non-
the reference period is one economic activities.
week. The NSSO defines following three broad
Which of the statements given Activity Status i) Working (engaged in an
above is/are correct? economic activity) i.e. ‘Employed’ ii)
(a) 1 only Seeking or available for work i.e.
(b) 2 only ‘Unemployed’ iii) Neither seeking nor
(c) Both 1 and 2 available for work.
(d) Neither 1 nor 2 All those individuals having a broad
activity status as i) or ii) above are
Answer: (c) classified as being in the Labour Force
Explanation: and those having activity status iii) are
Statement 1 is correct. classified as outside the Labour Force.
The National Sample Survey Thus, labour force constitutes of both
Organization (NSSO), since its inception employed and unemployed.
in 1950, does the measurement of Source:
employment / unemployment in India https://www.isid.ac.in/~planning/Unem
every 5 years. ployment2_w.pdf
Statement 2 is correct.
The National Sample Survey 25. Which of the following are
Organization (NSSO) provides three components of Pradhan Mantri
different estimates of employment and Awas Yojana?
unemployment based on different (1) In-situ rehabilitation of
approaches / reference periods used to existing slum dwellers using
classify an individual’s activity status. land as a resource through
These are the private participation
1. Usual status approach with a (2) Subsidy for Beneficiary-led
reference period of 365 days individual house
preceding the date of survey. construction/enhancement.
2. Current weekly status approach (3) Affordable Housing in
with a reference period of seven days Partnership
preceding the date of survey. Select the correct answer using the
3. Current daily status approach with code given below:
each day of the seven days (a) 1 and 2 only
preceding date of survey as the (b) 2 and 3 only
reference period. (c) 1 and 3 only
Additional Information: (d) 1, 2 and 3
In order to find out whether an individual
is employed or unemployed it needs to Answers: (d)

Prelim IAS Test Series (2019) – GS Test 10 (10.02.2019)


Economy 2 and Current Affairs Nov 2018 14
Explanation: (d) Neither 1 nor 2
All the given statements are correct.
Pradhan Mantri Awas Yojana (PMAY) is Answer: (b)
an initiative by Government of India in Explanation:
which affordable housing will be provided Statement 1 is incorrect.
to the urban and rural poor with a target On 29 May 2014, the Independent
of building 20 million affordable houses Evaluation Office, and not the 14th
by 2022. It has two components: Finance Commission submitted an
Pradhan Mantri Awas Yojana (Urban) assessment report to Prime Minister
(PMAY-U) for the urban poor and Narendra Modi with the recommendation
Pradhan Mantri Awas Yojana (Gramin) to replace the Planning Commission with
(PMAY-G and also PMAY-R) for the rural a “advisory commission” to promote co-
poor. operative federalism.
The Pradhan Mantri Awas Yojana will The Union Government of India
provide central assistance to Local announced the formation of NITI Aayog
Bodies and other implementing agencies on 1 January 2015, and the first meeting
through States/UTs for: was held on 8 February 2015.
 In-situ Rehabilitation of existing slum Statement 2 is correct.
dwellers using land as a resource The Fourteenth Finance Commission has
through private participation more than doubled the grant for local
 Credit Linked Subsidy bodies and recommended that nearly all
 Affordable Housing in Partnership of this money be spent on improving
 Subsidy for Beneficiary-led individual basic services.
house construction/enhancement. The Commission has divided the grant of
Credit linked subsidy component will be 2.87 lakh crore in two parts: A basic part
implemented as a Central Sector and a performance part. The
Scheme while other three components performance part would be dependent on
will be implemented as Centrally two factors. One, the local authority
Sponsored Scheme (CSS). would need to have an audited account
Source: https://pmaymis.gov.in/ for the previous year. Second, it will have
to demonstrate that it has increased its
26. Consider the following statements own revenue over the previous year.
regarding 14th Finance
Commission: 27. Regarding the 'Network for
(1) It recommended setting up of Development of Agricultural
NITI Aayog by replacing Cooperatives in Asia and the
Planning Commission to Pacific (NEDAC)' which was in the
promote cooperative news recently, consider the
federalism. following statements:
(2) It made provision of (1) NEDAC was set up in 1991
performance grant for by the United Nations’ Food
Panchayat Raj Institutions. and Agriculture Organisation
Which of the statements given (FAO), the International
above is/are correct? Cooperative Alliance (ICA)
(a) 1 only and the International Labour
(b) 2 only Organisation (ILO).
(c) Both 1 and 2

Prelim IAS Test Series (2019) – GS Test 10 (10.02.2019)


Economy 2 and Current Affairs Nov 2018 15
(2) The NEDAC is a regional (1) India was the world’s largest
forum linking cooperative oil consuming nation in 2018.
organisations of all the Asia- (2) In 2018, India was the fourth-
Pacific countries in promoting largest LNG importer in the
agricultural and rural world after Japan, South
development to ensure food Korea and China.
and livelihood security. Which of the statements given
(3) Union Agriculture Ministry above is/are correct?
and the National Cooperative (a) 1 only
Development Corporation (b) 2 only
(NCDC) are the members of (c) Both 1 and 2
NEDAC. (d) Neither 1 nor 2
Which of the statements given
above is/are correct? Answer: (b)
(a) 1 only Explanation:
(b) 1 and 2 only Statement 1 is incorrect and
(c) 1 and 3 only Statement 2 is correct.
(d) 1, 2 and 3 In 2018, India was the third-largest oil
consuming nation in the world and in the
Answer: (c) same year India was the fourth-largest
Explanation: Liquefied Natural Gas (LNG) importer
Statement 1 is correct. after Japan, South Korea and China.
Network for Development of Agricultural India has 0.5% of the oil and gas
Cooperatives in Asia and the Pacific resources of the world. India could
(NEDAC) was set up in 1991 by the overtake China as the world’s largest oil
United Nations’ Food and Agriculture demand growth centre by 2024,
Organisation (FAO), the International according to a Wood Mackenzie report.
Cooperative Alliance (ICA) and the
International Labour Organisation (ILO). 29. Consider the following statements
Statement 2 is incorrect. about 'food fortification and bio
The NEDAC sensitises governments in fortification':
the region on the role of agricultural (1) They are feasible and cost-
cooperatives in promoting agricultural effective methods of
and rural development to ensure food addressing SDG 1.
and livelihood security for millions of (2) Bio-fortification focuses on
people in Asia Pacific region. Its improving the nutritional
secretariat is at FAO Annex, Bangkok. content of the food during
Statement 3 is correct. plant growth while food-
Initially, NEDAC had 16 co-operative fortification is done manually
organisations from nine Asian countries before crop harvest.
as members. From India, Union (3) Both try to address the
Agriculture Ministry and NCDC were the hidden hunger.
initial members. At present, NEDAC has Which of the statements given
24-member organisations. above is/are correct?
(a) 1 only
28. Consider the following statements: (b) 1 and 3 only
(c) 3 only

Prelim IAS Test Series (2019) – GS Test 10 (10.02.2019)


Economy 2 and Current Affairs Nov 2018 16
(d) 1, 2 and 3 increased over the last 12 months to over
28 per cent in November 2018 as against
Answer: (c) 10.56 per cent in November 2017.
Explanation: Statement 2 is incorrect.
Statement 1 is incorrect. Revenue collection under GST is already
SDG 1 is about ending poverty. Food an area of worry, with Budget targets
fortification and bio fortification will help likely to be missed this year. The low
in SDG 2 in a concrete manner (End compliance rate could be one of the
hunger, achieve food security and possible roadblocks in extending the
improved nutrition) as compared to SDG composition scheme to service providers.
1. Under the composition scheme, traders,
Statement 2 is incorrect. manufacturers and restaurants pay a low
Bio-fortification focuses on improving the nominal tax rate.
nutritional content of the food during
plant growth while food-fortification is 31. Which of the following statements
done manually after the crop harvest. is/are correct regarding the
Statement 3 is correct. National Financial Reporting
Hidden hunger is related to micro- Authority (NFRA)?
nutrient deficiency, which is also a target (1) It is an independent regulator
under SDG 2. for the auditing profession
Source: Vajiram and Ravi Current and accounting standard.
Affairs booklet - November 2018, Page (2) It is a statutory body
4. replacing the Institute of
Chartered Accountants of
30. Consider the following statements India (ICAI).
with respect to Goods and Services Which of the statements given
Tax (GST) during the time period of above is/are correct?
Jan to Nov 2018: (a) 1 only
(1) The percentage of taxpayers, (b) 2 only
who have not filed returns (c) Both 1 and 2
has steadily increased. (d) Neither 1 nor 2
(2) Composition scheme has
been extended to service Answer. (a)
providers only and not to Explanation:
manufacturers. Statement 1 is correct.
Which of the statements given In the wake of accounting scams and
above is/are correct? frauds in the corporate sector, National
(a) 1 only Financial Reporting authority (NFRA)
(b) 2 only was notified as an independent regulator
(c) Both 1 and 2 for auditing profession and accounting
(d) Neither 1 nor 2 standard.
It consists of a Chairperson, three full
Answer: (a) time members and nine part-time
Explanation: members selected through a search-
Statement 1 is correct. cum-selection committee headed by
The percentage of taxpayers who have Cabinet Secretary.
not filed returns under GST has steadily

Prelim IAS Test Series (2019) – GS Test 10 (10.02.2019)


Economy 2 and Current Affairs Nov 2018 17
NFRA will review the quality of corporate
financial reporting in certain classes and Answer: (a)
subclasses of companies and take Explanation:
disciplinary action against auditors/audit The first Sustainable Blue Economy
firms for not discharging their statutory Conference was held in Nairobi, capital
duties with due diligence. of Kenya. It was organized by Kenya and
Statement 2 is incorrect. co-hosted by Japan and Canada. It
Section 132 of the Companies Act, 2013 builds on the momentum of the UN’s
provides for constitution of National 2030 Agenda for Sustainable
Financial Reporting Authority. The Development, the 2015 Climate Change
inherent regulatory role of ICAI as Conference in Paris and the UN Ocean
provided for in the Chartered Conference 2017 “Call to Action”.
Accountants Act, 1949 shall continue in
respect of its members in general and 33. Regarding the 'G-20 summits',
specifically with respect to audits consider the following statements:
pertaining to private limited companies, (1) India has never hosted any
and public unlisted companies below the G-20 summit.
threshold limit to be notified in the rules. (2) The first-ever G-20 summit
Value Addition: was held in Washington, DC.
The Quality Review Board (QRB) will (3) South America hosted the
also continue quality audit in respect of first ever G-20 summit in
private limited companies, public unlisted 2018.
companies below prescribed threshold Which of the statements given
and also with respect to audit of those above is/are correct?
companies that may be delegated to (a) 1 and 2 only
QRB by NFRA. (b) 3 only
Additional Info: (c) 2 and 3 only
The jurisdiction of NFRA for investigation (d) 1, 2 and 3
of Chartered Accountants and their firms
under section 132 of the Act would Answer: (d)
extend to listed companies and large Explanation:
unlisted public companies, the threshold The G20 (or Group of Twenty) is an
for which shall be prescribed in the rules. international forum for the governments
Source: and central bank governors from 19
http://pib.nic.in/newsite/PrintRelease.a countries and the European Union.
spx?relid=176918 Founded in 1999 with the aim to discuss
http://www.mca.gov.in/SearchableAct policy pertaining to the promotion of
s/Section132.htm international financial stability, the G20
has expanded its agenda since 2008 and
32. The first global 'Sustainable Blue heads of government or heads of state,
Economy Conference' was held as well as finance ministers and foreign
recently in which of the following ministers, have periodically conferred at
place? summits ever since. It seeks to address
(a) Nairobi, Kenya issues that go beyond the responsibilities
(b) New Delhi, India of any one organization.
(c) Tokyo, Japan Statement 1 is correct.
(d) Ottawa, Canada

Prelim IAS Test Series (2019) – GS Test 10 (10.02.2019)


Economy 2 and Current Affairs Nov 2018 18
India will host the first G-20 summit in the information about an entity’s ownership
year 2022. India has never hosted any structure.
G-20 summit earlier. Statement 1 is incorrect.
Statement 2 is correct. It is conceived by G20, on the
On November 16-17, 2008, in recommendation of report by (Financial
Washington, DC, the G-20 held it’s first- Stability Board) FSB in the wake of
ever summit. Before this meeting, the G- global financial crisis which highlighted
7 guided most global economic plans. significant obstacles to identifying and
Statement 3 is correct. tracing financial transactions across the
The 2018, G-20 Summit has held in the international financial system.
city of Buenos Aires, Argentina. This is Statement 2 is correct.
the first G-20 summit which was held in Globally, it is implemented and
South America. maintained by Global Legal Entity
Identifier Foundation (GLEIF) through
34. The term ‘Legal Entity Identifier Local Operation Units (LOU) established
(LEI) code’ recently was by each country independently and
sometimes seen in news. Consider voluntarily.
the following statements with In January 2013, the LEI Regulatory
reference to it: Oversight Committee (ROC) took over
(1) It is developed by World the responsibility for development and
Bank which served as a implementation of the Global LEI
global directory enhancing System. The FSB established the GLEIF
the transparency in the as a non-for-profit foundation in 2014.
global marketplace. In India entities can obtain LEI from
(2) LEI is implemented and Legal Entity Identifier India Ltd (LEIL)
maintained by Global Legal (only LOU of India), subsidiary of The
Entity Identifier Foundation. Clearing Corporation of India Ltd,
(3) It is mandatory for all market recognized by the Reserve Bank of India
participants both entities as (RBI) under the Payment and Settlement
well as individuals. Systems Act, 2007.
Which of the statements given Statement 3 is incorrect.
above is/are correct? Recently, the RBI has made LEI code
(a) 1 and 2 only mandatory for all market participants
(b) 2 only except individual. As per the RBI,
(c) 2 and 3 only banks are required to make it mandatory
(d) 1, 2 and 3 for corporate borrowers having
aggregate fund-based and non-fund-
Answer: (b) based exposure of ₹5 crore and above
Explanation: from any bank to obtain LEI registration
The Legal Entity Identifier (LEI) is a 20- and capture the same in the Central
character, alpha-numeric code based on Repository of Information on Large
the ISO 17442 standard. It connects to Credits (CRILC).
key reference information that enables Source:
clear and unique identification of legal https://www.gleif.org/en/about-
entities participating in financial lei/introducing-the-legal-entity-
transactions. Each LEI contains identifier-lei

Prelim IAS Test Series (2019) – GS Test 10 (10.02.2019)


Economy 2 and Current Affairs Nov 2018 19
https://www.thehindubusinessline.co of doing business index. A credit
m/money-and-banking/ask-corporate- repository helps banks distinguish
borrowers-to-obtain-legal-entity- between a bad and a good borrower and
identifier-code-rbi-tells- accordingly offer attractive interest rates
banks/article9939975.ece to good borrowers and higher interest
rates to bad borrowers.
35. With respect to Public Credit Option (d) is incorrect.
Registry (PCR) being setup by Data on borrowings from banks, non-
RBI, identify the incorrect banking financial companies, corporate
statement from the following: bonds or debentures from the market,
(a) By having a registry of all external commercial borrowings (ECBs),
loans in the form of a Public foreign currency convertible
Credit Registry, the credit bonds(FCCBs), masala bonds, and inter-
delivery system can be made corporate borrowings are not available in
more efficient, furthering one data repository. PCR will help
financial inclusion. capture all relevant information about a
(b) It can address the bad loan borrower, across different borrowing
problem with Indian banks, as products in one place. It can flag early
corporate debtors will not be warnings on asset quality by tracking
able to borrow across banks performance on other credits.
without disclosing existing Source: Vajiram and Ravi Current
debt. Affairs booklet - November 2018, Page
(c) Public Credit Registry may 24
help to improve the ranking of
India in the World Bank’s 36. In the context of the 'Global
Ease of Doing Business Compact for Migration', sometimes
Index. seen in the news, consider the
(d) The Public Credit Registry following statements:
will collect the information (1) It is the first ever inter-
from the banking entities governmentally negotiated
only. agreement, prepared under
the auspices of the United
Answer: (d) Nations.
Explanation: (2) The Compact is an
Option (a) is correct. international treaty and will
PCR will address issues such as be binding under the
information asymmetry, improve access international law.
to credit and strengthen the credit culture Which of the statements given
among consumers. above is/are correct?
Option (b) is correct. (a) 1 only
It can also address the bad loan problem (b) 2 only
staring at banks, as corporate debtors (c) Both 1 and 2
will not be able to borrow across banks (d) Neither 1 nor 2
without disclosing existing debt.
Option (c) is correct. Answer: (a)
Setting up the PCR will help improve Explanation:
India’s rankings in the World Bank’s ease Statement 1 is correct.

Prelim IAS Test Series (2019) – GS Test 10 (10.02.2019)


Economy 2 and Current Affairs Nov 2018 20
Global Compact for Migration is the first, (1) If the ToT of a country
intergovernmental negotiated agreement, increases, the country
prepared under the auspices of the benefits.
United Nations, to cover all dimensions (2) If the ToT of a country
of international migration in a holistic and decreases, the country stands
comprehensive manner. to lose.
In September 2016 the General Which of the statements given
Assembly decided, through the adoption above is/are correct?
of the New York Declaration for (a) 1 only
Refugees and Migrants, to develop a (b) 2 only
global compact for safe, orderly and (c) Both 1 and 2
regular migration. (d) Neither 1 nor 2
Statement 2 is incorrect.
The compact was formally endorsed by Answer: (c)
the United Nations General Assembly on Explanation:
19 December 2018. As the Compact is ToT = Price of exports/ Price of Imports
not an international treaty, it will be non- for a country.
binding under international law.
39. Regarding 'Minor Forest Produce
37. The CriSidEx index is seen in news (MFP)', sometimes seen in the
sometimes. This index is related to news, consider the following
which of the following? statements:
(a) House prices. (1) MFP does not come under
(b) Outlook on MSME sector. the government’s minimum
(c) Banking sector performance. support price scheme.
(d) Power sector. (2) Under the Panchayats
(Extension to the Scheduled
Answer: (b) Areas) Act, 1996, gram
Explanation: sabha has the power to
CriSidEx is India’s first sentiment index regulate and restrict the
jointly developed by CRISIL and SIDBI. It ownership of MFP.
is a composite index based on the Which of the statements given
diffusion index of 8 parameters and above is/are correct?
measures MSME business sentiment on (a) 1 only
a scale of 0 to 200. The crucial benefit of (b) 2 only
CriSidEx is that its readings will flag (c) Both 1 and 2
potential headwinds and changes in (d) Neither 1 nor 2
production cycles and help improve
market efficiencies in MSME sector. And Answer: (b)
by capturing the sentiments of exporters Explanation:
and importers, it will also offer actionable Minor Forest Produce (MFP) is a subset
indicators on foreign trade. of forest produce and got a definition only
in 2007 when the Scheduled Tribes and
38. Consider the following statements Other Traditional Forest Dwellers
regarding “Terms of Trade (ToT)” (Recognition of Forest Rights) Act, 2006,
used in the field of international was enacted. Section 2(i) of the said Act
trade: defines a Minor Forest Produce (MFP) as

Prelim IAS Test Series (2019) – GS Test 10 (10.02.2019)


Economy 2 and Current Affairs Nov 2018 21
all non-timber forest produce of plant additional forest and tree
origin and includes bamboo, brushwood, cover by 2030.
stumps, canes, Tusser, cocoon, honey,
waxes, Lac, tendu/kendu leaves, Answer: (c)
medicinal plants and herbs, roots, tuber Explanation:
and the like. Statements in options (a), (b) and (d)
Statement 1 is correct. are correct.
The Government of India has launched a INDCs are declaration of individual
central sector scheme for marketing of countries which indicate what post-2020
Minor Forest Produce through Minimum climate actions they intend to take under
Support Price (MSP) and development of a new international agreement, known as
value chain to ensure fair monetary their Intended Nationally Determined
returns to MFP gatherers for their efforts Contributions (INDCs).
in collection, primary processing, Option (c) is incorrect.
storage, packaging, transportation etc. To achieve about 40 per cent cumulative
The scheme envisages fixation and electric power installed capacity from
declaration of Minimum Support Price for non-fossil fuel-based energy resources
the selected MFP based on the by 2030, with the help of transfer of
suggestions /inputs received from Tribal technology and low-cost international
Cooperative Marketing Development finance, including from Green Climate
Federation of India (TRIFED) which Fund.
came into existence in 1987.
Statement 2 is correct. 41. Which one of the following is the
The Panchayats (Extension to the best description of ‘Economic
Scheduled Areas) PESA Act, 1996 Capital Framework’ that was in the
empowers the gram sabha to regulate news recently?
and restrict the ownership of MFP. (a) Through this framework
banks borrow from Reserve
40. Which of the following is not a Bank of India in emergency
salient feature of ‘India’s Intended situation when inter-bank
Nationally Determined liquidity dries up completely.
Contributions’? (b) The framework decides the
(a) Propagate a healthy and rate at which the RBI lends
sustainable way of living short-term money to the
based on traditions and banks against securities.
values of conservation and (c) The framework allows the
moderation. banks in India to hold a
(b) Reduce the emissions certain proportion of their
intensity of its GDP by 33 to deposits in the form of cash.
35% by 2030 from 2005 level. (d) The framework refers to the
(c) Achieve 100% electric power risk capital required by the
installed capacity from non- central bank while taking into
fossil fuel-based energy account different risks.
resources by 2030.
(d) Create an additional carbon Answer: (d)
sink of 2.5 to 3 billion tonnes Explanation:
of CO2 equivalent through

Prelim IAS Test Series (2019) – GS Test 10 (10.02.2019)


Economy 2 and Current Affairs Nov 2018 22
Economic Capital Framework refers to news, consider the following
the risk capital required by the central statements:
bank while taking into account different (1) The United Nations
risks. The economic capital framework Framework Convention on
reflects the capital that an institution Climate Change (UNFCCC)
requires or needs to hold as a counter in association with the World
against unforeseen risks or events or Bank has released the Index.
losses in the future. (2) The Index has been
The Reserve Bank of India (RBI), in constructed by taking all the
consultation with the government, has 17 SDGs into consideration.
set up a six-member Committee to (3) According to the Index,
review the economic capital framework of Maharashtra, Gujarat and
the central bank. The former RBI Himachal Pradesh are on
Governor Bimal Jalan will be the track to achieve the SDGs on
Committee’s Chairman and former time.
Deputy Governor Rakesh Mohan the Which of the statements given
Deputy Chairman. above is/are incorrect?
(a) 1 and 2 only
42. Which one of the following (b) 3 only
countries has launched the world’s (c) 1, 2 and 3
first Sovereign Blue Bond, a (d) None of the above
financial instrument designed to
support sustainable marine and Answer: (c)
fisheries projects? Explanation:
(a) India Statement 1 is incorrect.
(b) Seychelles The NITI Aayog has recently released
(c) Maldives the Baseline Report of the Sustainable
(d) Fiji Development Goals (SDG) India Index,
which comprehensively documents the
Answer: (b) progress made by India’s States and
Explanation: Union Territories towards implementing
The Republic of Seychelles has launched the 2030 SDG targets.
the world’s first Sovereign Blue Bond, a Statement 2 is incorrect.
financial instrument designed to support The Index has been constructed
sustainable marine and fisheries spanning across 13 out of 17 SDGs
projects. With this, Seychelles became (leaving out Goals 12, 13, 14 and 17).
the first nation to pioneer such a novel Statement 3 is incorrect.
financing instrument. The bond raised According to the SDG Index, Himachal
USD 15 million from international Pradesh, Kerala, and Tamil Nadu have
investors. The bond demonstrates the been ranked on top 3 positions and are
potential for countries to harness capital on track to achieve the UN Sustainable
markets for financing the sustainable use Development Goals (SDG).
of marine resources.
44. With reference to National Nutrition
43. In the context of the 'Sustainable Mission (NNM), consider the
Development Goals (SDGs) India following:
Index', sometimes seen in the

Prelim IAS Test Series (2019) – GS Test 10 (10.02.2019)


Economy 2 and Current Affairs Nov 2018 23
(1) This Mission is a joint effort Source:
of the Ministry of Health and http://pib.nic.in/newsite/PrintRelease.asp
Family Welfare and the x?relid=174025
Ministry of Women and Child https://pibindia.wordpress.com/tag/nation
Development (WCD) al-nutrition-mission/
(2) It would strive to achieve
reduction in stunting to 25% 45. Which of the following are the
by 2022. indicators used by World Economic
(3) Under this Mission only the Forum to compute the Gender Gap
backwards districts will be Index?
covered. (1) Economic opportunity
Which of the statements given (2) Social status
above is/are correct? (3) Educational attainment
(a) 1 and 2 only (4) Political Empowerment
(b) 1 and 3 only Select the correct answer using the
(c) 2 and 3 only code given below:
(d) 1, 2 and 3 (a) 1, 2 and 3 only
(b) 2 and 4 only
Answers: (a) (c) 1, 3 and 4 only
Explanation: (d) 1, 2, 3 and 4
Statement 1 is correct.
NNM is a joint effort of Ministry of Health Answer: (c)
and Family Welfare and the Ministry of Explanation:
Women and Child development (WCD) India made no improvement in the overall
towards a life cycle approach for gender gap ranking released by the
interrupting the intergenerational cycle of World Economic Forum (WEF) in 2018,
under nutrition. compared to 2017. It stood at a low 108
Statement 2 is correct. out of the 149 countries in 2018, the
NNM will be rolled out in three phases same as in 2017. Global gender gap
from 2017-18 to 2019-20. NNM targets to report is published annually by the world
reduce stunting, under-nutrition, anaemia economic forum since 2006. Global
(among young children, women and gender gap index is a part of this which
adolescent girls) and reduce low birth measures gender equality across four
weight by 2%, 2%, 3% and 2% per pillars– they are economic opportunity,
annum respectively. Although the target political empowerment, educational
to reduce Stunting is at least 2% p.a., attainment and health and survival.
Mission would strive to achieve reduction
in Stunting from 38.4% (NFHS-4) to 25% 46. Consider the following statements
by 2022 (Mission 25 by 2022). about European Union (EU):
Statement 3 is incorrect. (1) The European Union is the
More than 10 crore people will be second largest trade bloc in
benefitted by this programme. All the the world after ASEAN.
States and districts will be covered in a (2) The member States of EU
phased manner i.e. 315 districts in 2017- remain sovereign and
18, 235 districts in 2018-19 and independent states and they
remaining districts in 2019-20. have decided to pool some of
their 'sovereignty' in EU.

Prelim IAS Test Series (2019) – GS Test 10 (10.02.2019)


Economy 2 and Current Affairs Nov 2018 24
(3) All EU member States are 2004, 2007 and 2013, after the Euro was
legally committed to joining launched in 2002.
the Euro, with the exception Countries with an opt-out: Concerning
of Denmark and Britain. the single currency, this is the case for
(4) All the members of EU use the United Kingdom and Denmark.
Euro as their official national Both these countries kept their currency
currency. after becoming members of the EU.
Which of the statements given
above is/are correct? 47. Which of the following statements
(a) 1 and 4 only is/are correct regarding the
(b) 1 and 3 only ‘Partners’ Forum’, sometimes seen
(c) 2 and 3 only in the news?
(d) 2 and 4 only (1) It is a global partnership to
achieve the free movements
Answer: (c) of goods and labour.
Explanation: (2) The partnership is an alliance
Statement 1 is incorrect. of all the member countries
The Union currently counts 28 EU of the WTO.
countries (including UK for the time being (3) India has hosted the fourth
as of Feb 2019) and is the largest trade Partners’ Forum in
bloc in the world. association with the WTO.
With just 6.9 % of the world's population, Which of the statements given
EU trade with the rest of the world above is/are incorrect?
accounts for some 15.6 % of global (a) 1 only
imports and exports. (b) 1 and 2 only
Together with the United States and (c) 1, 2 and 3
China, the EU is one of the 3 largest (d) None of the above
global players in international trade.
Statement 2 is correct. Answer: (c)
The member States of EU remain Explanation:
sovereign and independent states and Statement 1 is incorrect.
they have decided to pool some of their Partners’ Forum is a global health
'sovereignty' in EU. Member states can partnership launched in September 2005
negotiate an opt-out from any of the to accelerate efforts to reduce child and
European Union legislation or treaties, maternal mortality, improve adolescent,
and agree to not participate in certain child, newborn and maternal health.
policy areas. Statement 2 is incorrect.
Statements 3 and 4 are incorrect. This partnership is an alliance of more
The euro (€) is the official currency of 19 than 1,000 plus members, across 10
out of 28 EU countries. These countries constituencies in 92 countries: academic,
are collectively known as the Eurozone. research and teaching institutions;
Non-euro area countries: These are donors and foundations; health care
countries where the Euro has still not professionals; multilateral agencies; non-
been adopted, but who will join once they governmental organizations; partner
have met the necessary conditions. countries; global financing mechanisms
Mostly, it consists of countries of member and the private sector.
states which acceded to the Union in Statement 3 is incorrect.

Prelim IAS Test Series (2019) – GS Test 10 (10.02.2019)


Economy 2 and Current Affairs Nov 2018 25
The fourth Partners’ Forum was recently Project Sashakt is a five-pronged
held in New Delhi. It was hosted by the strategy towards resolution of
Government of India, in association with stressed assets, as recommended by
the Partnership for Maternal, New-born Sunil Mehta Committee which are:
and Child Health (PMNCH). The previous  Small and Medium Enterprise
chapters were held in Johannesburg, resolution approach
South Africa (2014), New Delhi, India  Bank led resolution approach
(2010) and Dar Es Salaam, Tanzania  Asset Management Company /
(2007). This is the second time India is Alternative Investment Fund
hosting the Partners’ Forum.  NCLT/ IBC approach
 Asset-trading approach
48. Consider the following statements Source:
about ‘Project Sashakt’: https://www.thehindubusinessline.co
(1) It is a comprehensive plan for m/money-and-banking/mehta-panel-
the resolution of stressed submits-5-point-plan-to-fight-
assets in the banking sector. npas/article24314726.ece
(2) It involves pro-active
government interference. 49. For nations facing domestic
(3) Sunil Mehta Committee financial crisis, a term ‘Dutch
recommended five-pronged Disease’ is used sometimes. Which
strategy towards resolution of among the following is not its
stressed assets. characteristic?
Which of the statements given (a) The negative consequences
above is/are correct? arising from large increases
(a) 1 only in the value of a country's
(b) 1 and 3 only currency.
(c) 2 and 3 only (b) A substantial increase in
(d) 1, 2 and 3 natural resource prices.
(c) Any development that results
Answer: (b) in a large inflow of foreign
Explanation: currency.
Statement 1 is correct. (d) None of the above.
Recently, the Government of India
announced a comprehensive plan Answer: (d)
Project Sashakt for the resolution of Explanation:
stressed assets in the banking sector. It All the given statements describe the
aims to strengthen the credit capacity, characteristics of Dutch Disease.
credit culture and credit portfolio of public The term ‘Dutch disease’ refers to the
sector banks. deindustrialization of a nation's economy
Statement 2 is incorrect. that occurs when the discovery of a
The plan does not involve government natural resource raises the value of that
interference as it would entirely be led nation's currency, making manufactured
by the banks. Also, it does not require goods less competitive with other
any law to be enacted. All provisions nations; increasing imports and
comply with existing regulation of decreasing exports. The term originated
banking sector. in Holland after the discovery of North
Statement 3 is correct. Sea gas.

Prelim IAS Test Series (2019) – GS Test 10 (10.02.2019)


Economy 2 and Current Affairs Nov 2018 26
It also refers to large inflow of foreign
currency resulting from FDI or foreign 51. In the context of the ‘Pradhan
assistance. Mantri Matru Vandana Yojana
Source: Vajiram and Ravi Current (PMMVY)’, consider the following
Affairs booklet - November 2018, Page statements:
40 (1) It is implemented in all the
districts of the country in
50. Which of the following countries accordance with the
are not the members of the provisions of the National
Organization of the Petroleum Food Security Act, 2013.
Exporting Countries (OPEC), the (2) It provides partial
world's leading oil producing bloc? compensation for the wage
(1) Ecuador loss in terms of cash
(2) Algeria incentives so that the woman
(3) Indonesia can take adequate rest
(4) Nigeria before and after delivery of
(5) Qatar the first living child.
Select the correct answer using the (3) The focal point of
code given below: implementation of the
(a) 2, 3 and 5 only scheme is the Anganwadi
(b) 3 and 5 only Centre and ASHA / ANM
(c) 2 and 3 only workers.
(d) 1, 3, 4 and 5 Which of the statements given
above is/are correct?
Answer: (b) (a) 1 and 2 only
Explanation: (b) 3 only
OPEC was founded in 1960 by Iran, Iraq, (c) 1, 2 and 3
Kuwait, Saudi Arabia and Venezuela, five (d) None of the above
major oil-producing countries. Currently,
the Organization has a total of 15 Answer: (c)
Member Countries. The current OPEC Explanation:
members are the following: Algeria, Pradhan Mantri Matru Vandana Yojana
Angola, Ecuador, Equatorial Guinea, (PMMVY) is a Centrally Sponsored
Gabon, Iran, Iraq, Kuwait, Libya, Nigeria, Scheme implemented by Ministry of
the Republic of the Congo, Saudi Arabia, Women and Child Development.
United Arab Emirates, and Venezuela. Statement 1 is correct.
Indonesia was a former member; Qatar The scheme is being implemented on a
is no longer a member of OPEC starting 60:40 cost-sharing basis with the State
from 1 January 2019. Qatar is not the governments. Maternity Benefit
first country to leave OPEC. Indonesia, Programme would be implemented in all
Gabon and Ecuador have all previously the districts of the country in accordance
left. All three returned at a later stage, with the provision of the National Food
but Indonesia, which re-joined and Security Act, 2013.
withdrew a second time in 2016, has not Statement 2 is correct.
returned to the bloc since. After Qatar's The objectives of the scheme are:
decision to leave the bloc, 14-member providing partial compensation for the
countries remain. wage loss in terms of cash incentives so

Prelim IAS Test Series (2019) – GS Test 10 (10.02.2019)


Economy 2 and Current Affairs Nov 2018 27
that the woman can take adequate rest The Global Carbon Project works
before and after delivery of the first living collaboratively with the International
child; and the cash incentives provided Geosphere - Biosphere Programme, the
would lead to improved health seeking World Climate Programme, the
behaviour amongst the Pregnant Women International Human Dimensions
and Lactating Mothers. Programme on Global Environmental
Statement 3 is correct. Change and Diversities, under the Earth
PMMVY is implemented using the System Science Partnership.
platform of Anganwadi Services scheme Statement 2 is correct.
of Umbrella ICDS under Ministry of The Global Carbon Atlas is established
Women and Child Development. PMMVY by the Global Carbon project.
shall be implemented through a centrally
deployed Web Based MIS Software 53. With reference to the 'Climate
application and the focal point of Vulnerable Forum (CVF)',
implementation would be the Anganwadi sometimes seen in the news,
Centre (AWC) and ASHA/ ANM workers. consider the following statements:
(1) The CVF is an international
52. Recently the 'Global Carbon cooperation group of
Project' was in the news. In this developing countries tackling
context, consider the following global climate change.
statements: (2) The CVF was set up by India
(1) It is a project under the in the Global Climate Action
Global Energy Parliament. Summit held in San
(2) The Global Carbon Atlas is Francisco, California.
established by the Global (3) The United Nation’s agencies
Carbon Project. collaborate in implementing
Which of the statements given activities linked to the CVF.
above is/are correct? Which of the statements given
(a) 1 only above is/are correct?
(b) 2 only (a) 1 only
(c) Both 1 and 2 (b) 2 and 3 only
(d) Neither 1 nor 2 (c) 1 and 3 only
(d) 1, 2 and 3
Answer: (b)
Explanation: Answer: (c)
The Global Carbon Project was formed in Explanation:
2001 to help the international science Statement 1 is correct.
community to establish a common, The Climate Vulnerable Forum (CVF) is
mutually agreed knowledge base that an international cooperation group of
supports policy debate and action to slow developing countries tackling global
the rate of increase of greenhouse gases climate change.
in the atmosphere. It is a Global Statement 2 is incorrect.
Research Project of Future Earth and a The CVF was set up by the Maldives
research partner of the World Climate government before the 2009 United
Research Programme. Nations Climate Change Conference in
Statement 1 is incorrect. Copenhagen, which sought to increase

Prelim IAS Test Series (2019) – GS Test 10 (10.02.2019)


Economy 2 and Current Affairs Nov 2018 28
awareness of countries considered Statement 2 is incorrect.
vulnerable. The Himalayan State Regional Council
Statement 3 is correct. will consist of - Chairman- NITI Aayog
United Nations agencies collaborate in member, Chief Secretaries of the
implementing activities linked to the CVF Himalayan States, Secretaries of key
with the UNDP, the lead organization Central Ministries, senior officers of NITI
supporting the forum’s work. The CVF Aayog and Special invitees.
was formed to increase the accountability
of industrialized nations for the 55. With reference to the 'East Asia
consequences of global climate change. Summit' (EAS), sometimes seen in
Afghanistan, Nepal and Bhutan are its the news, consider the following
members, whereas India is one of the statements:
observer states. (1) It is a unique Leaders-led
forum of 18 countries of the
54. In the context of the Himalayan Asia-Pacific.
State Regional Council (HSRC), (2) All the ASEAN and the
which was in the news recently, SAARC countries are the
consider the following statements: members of EAS.
(1) HSRC will be the nodal (3) EAS is an initiative of ASEAN
agency for the Sustainable and is based on the premise
development in the of the centrality of ASEAN.
Himalayan Region consisting Which of the statements given
of 12 States. above is/are correct?
(2) The Chairman of the HSRC (a) 1 and 2 only
will be the Prime Minister of (b) 1 and 3 only
India. (c) 1, 2 and 3
Which of the statements given (d) None of the above
above is/are correct?
(a) 1 only Answer: (b)
(b) 2 only Explanation:
(c) Both 1 and 2 Statement 1 is correct.
(d) Neither 1 nor 2 The East Asia Summit is a unique
Leaders-led forum of 18 countries of the
Answer: (a) Asia-Pacific region formed to further the
Explanation: objectives of regional peace, security and
Statement 1 is correct. prosperity. It has evolved as a forum for
NITI Aayog has constituted Himalayan strategic dialogue and cooperation on
State Regional Council (HSRC) for the political, security and economic issues of
Sustainable Development of Indian common regional concern and plays an
Himalayan Region. The HSRC will be the important role in the regional
nodal agency for the Sustainable architecture. Established in 2005, EAS
development in the Himalayan Region allows the principal players in the Asia-
consisting 12 States of India. It will Pacific region to discuss issues of
monitor the implementation of action common interest and concern, in an
points for Central Ministries, Institutions open and transparent manner, at the
and Himalayan State Governments in highest level.
Indian Himalayan Region. Statement 2 is incorrect.

Prelim IAS Test Series (2019) – GS Test 10 (10.02.2019)


Economy 2 and Current Affairs Nov 2018 29
The membership of EAS consists of ten
ASEAN Member States (i.e. Brunei 57. Regarding the 'National Super-
Darussalam, Cambodia, Indonesia, Lao computing Mission (NSM)', which
PDR, Malaysia, Myanmar, Singapore, was in the news recently, consider
Thailand, the Philippines and Vietnam), the following statements:
Australia, China, India, Japan, New (1) The objective of the mission
Zealand, Republic of Korea, Russian is to develop the world's
Federation and the USA. fastest supercomputer and to
Statement 3 is correct. make the country self-reliant
EAS is an initiative of ASEAN and is in the computer technology.
based on the premise of the centrality of (2) The Mission supports the
ASEAN. government's vision of
'Digital India' and 'Make in
56. In the context of the 'Gulf India' initiatives.
Cooperation Council (GCC)’, (3) The mission is implemented
sometimes seen in the news, by the Department of
consider the following statements: Science and Technology and
(1) The mission of the Council is the Department of
to coordinate and unify the Electronics and Information
petroleum policies among its Technology.
member countries. Which of the statements given
(2) Yemen and Jordan are not above is/are correct?
the members of the Gulf (a) 1 only
Cooperation Council. (b) 1 and 2 only
Which of the statements given (c) 2 and 3 only
above is/are correct? (d) 1, 2 and 3
(a) 1 only
(b) 2 only Answer: (c)
(c) Both 1 and 2 Explanation:
(d) Neither 1 nor 2 Statement 1 is incorrect.
National Super-computing Mission
Answer: (b) (NSM) envisages empowering national
Explanation: academic and R&D institutions spread
Statement 1 is incorrect. over the country by installing a vast
The GCC was established in Riyadh, super-computing grid comprising of more
Saudi Arabia, in May 1981. The purpose than 70 high-performance computing
of the GCC is to achieve unity among its facilities.
members based on their common Statement 2 is correct.
objectives and their similar political and The mission supports the government's
cultural identities, which are rooted in mission of Digital India and Make in India
Arab and Islamic cultures. initiatives.
Statement 2 is correct. Statement 3 is correct.
Gulf Cooperation Council (GCC), political The mission would be implemented by
and economic alliance of six Middle the Department of Science and
Eastern countries - Saudi Arabia, Kuwait, Technology and Department of
the United Arab Emirates, Qatar, Electronics and Information Technology
Bahrain, and Oman. (DeitY) through Centre for Development

Prelim IAS Test Series (2019) – GS Test 10 (10.02.2019)


Economy 2 and Current Affairs Nov 2018 30
of Advanced Computing (C-DAC) and
Indian Institute of Science (IISc), Answer: (c)
Bangalore. This will be done at an Explanation:
estimated cost of Rs 4,500 crore over a Statement 1 is correct.
period of seven years. Momentum for Change is an initiative
spearheaded by the UN Climate Change
58. 'Regulatory Indicators for secretariat to shine a light on the
Sustainable Energy (RISE)', report enormous groundswell of activities
is released by which of the underway across the globe that are
following? moving the world toward a highly
(a) International Energy Agency resilient, low-carbon future.
(b) World Bank Statement 2 is correct.
(c) International Energy Forum Momentum for Change recognizes
(d) World Economic Forum innovative and transformative solutions
that address both climate change and
Answer: (b) wider economic, social and
Explanation: environmental challenges. The Indian
World Bank has released its report - private project, HelpUsGreen is one of 15
Regulatory Indicators for Sustainable ground-breaking projects from around
Energy (RISE) 2018 - charting global the world that has won this year’s UN
progress on sustainable energy policies. climate action award.
The report was released on the side-lines
of the 24th Conference of the Parties to 60. Which of the following is/are
the UN Framework Convention on agreed in the 2018 United Nations
Climate Change(COP24). According to Climate Change Conference (COP-
the report India has gained a great 24) that took place in Katowice,
success in renewable energy auctions Poland?
that delivered record-setting low prices (1) Rules to implement the Paris
for solar power. Agreement, which will come
into force and replace the
59. What is/are the 'Momentum for existing Kyoto Protocol in
Change', which was in the news 2020.
recently? (2) Establishing new climate
(1) It is an initiative by the UN finance targets from 2025
Climate Change secretariat. onwards to follow on from the
(2) It recognizes innovative and target of mobilizing $100
transformative solutions that billion a year from 2020 to
address both climate change support developing countries.
and wider economic, social Select the correct answer using the
and environmental code given below:
challenges. (a) 1 only
Select the correct answer using the (b) 2 only
code given below: (c) Both 1 and 2
(a) 1 only (d) Neither 1 nor 2
(b) 2 only
(c) Both 1 and 2 Answer: (c)
(d) Neither 1 nor 2 Explanation:

Prelim IAS Test Series (2019) – GS Test 10 (10.02.2019)


Economy 2 and Current Affairs Nov 2018 31
The 2018 United Nations Climate (c) 2, 3 and 4 only
Change Conference that took place (d) 2 and 3 only
between 2 and 15 December 2018 in
Katowice, Poland, is the 24th Conference Answer: (c)
of the Parties to the United Nations Explanation:
Framework Convention on Climate Banks prefer to never write off bad debt
Change (COP24). It is also referred to as since their loan portfolios are their
the Katowice Climate Change primary assets and source of future
Conference or Katowice Climate Talks. revenue. However, toxic loans, or loans
The most important outcome of COP24 that cannot be collected, reflect very
was that the countries have agreed on poorly on a bank’s financial statement.
rules for the implementation of the 2015
Paris Agreement. The Katowice package 62. Consider the following statements
includes guidelines that will regarding the Price Deficiency
operationalize the transparency Payment (PDP) scheme:
framework. It sets out how countries will (1) Under this scheme the
provide information about their Nationally government pays the
Determined Contributions (NDCs) that difference between Minimum
describe their domestic climate actions. Support Price (MSP) and
This information includes mitigation and market price of an
adaptation measures as well as details of agricultural product.
financial support for climate action in (2) This scheme can do away
developing countries. Besides with handling and logistic
transparency framework, the Katowice cost of agricultural goods.
package also includes guidelines that (3) It will go against the WTO
relate to the process for establishing new norm on subsidy restriction.
targets on finance from 2025 onwards to Which of the statements given
follow-on from the current target of above is/are correct?
mobilizing $100 billion per year from (a) 1 and 2 only
2020 to support developing countries. (b) 1 and 3 only
(c) 2 and 3 only
61. When a bank writes off a loan it (d) 1, 2 and 3
implies that:
(1) The bank gives waiver on Answer: (a)
that loan. Explanation:
(2) The bank declassifies that Under Price Deficiency Payment
loan as an asset. scheme, farmers are proposed to be
(3) The bank may transfer this compensated for the difference between
loan to Asset Reconstruction the government-announced MSPs for
Company to recover. select crops and their actual market
(4) It gives a more accurate and prices. The key benefit from the price
objective picture of financial deficiency payment is that it will reduce
health of the bank. the need for the government to actually
Which of the statements given procure food crops, transport and store
above is/are correct? them and then dispose of them under
(a) 1 only PDS. The difference between the support
(b) 1 and 3 only and market prices can instead simply be

Prelim IAS Test Series (2019) – GS Test 10 (10.02.2019)


Economy 2 and Current Affairs Nov 2018 32
paid in cash to the farmer. Price sector in 2017, which will ease out fund
deficiency payment scheme can also related and another logistics bottleneck.
keep India’s bill on food subsidies under Statement 3 is correct.
check because of reduction in Government has launched Logistic Ease
implementation cast and better targeting. Across Different States (LEADS) in order
In similar way EU was able to comply the to look at logistics bottleneck at state
WTO norm. level.

63. With reference to Logistics sector, 64. Consider the following statements
which of the statements given about Concession Financing
below is/are correct? Scheme:
(1) Logistic Performance Index is (1) It is aimed to support Indian
released by the World Bank entities bidding for
bi-annually. strategically important
(2) Logistic Sector has been infrastructure projects
given infrastructure status by abroad.
the Government of India. (2) Under the scheme Ministry of
(3) LEADS Index has been External Affairs selects the
launched as a domestic projects
index. Which of the above given
Select the correct answer using the statements is/are correct?
code given below: (a) 1 only
(a) 1 and 3 only (b) 2 only
(b) 2 and 3 only (c) Both 1 and 2
(c) 3 only (d) Neither 1 nor 2
(d) 1, 2 and 3
Answer: (c)
Answer: (d) Explanation:
Explanation: Statement 1 is correct.
Statement 1 is correct. Concession Financing Scheme is
Logistic Performance Index is aimed to support Indian entities
released by the World Bank bi- bidding for strategically important
annually by comparing across 160 infrastructure projects abroad. It will
countries on logistic sector performance. now cover all Indian entities, compared to
It analyses countries through six the earlier stipulation of minimum 75%
indicators; Indian shareholding. Recently, the
 Custom government extended the Concession
 International shipment Financing Scheme for five years till
 Tracking and trace 2023.
 Infrastructure Statement 2 is correct.
 Competence Under the scheme Ministry of External
 Timeliness of consignment Affairs selects the projects, keeping in
Statement 2 is correct. view the strategic interest of India and
As an initiative, Government of India sends the same to the Department of
gave infrastructure status to logistic Economic Affairs.

Prelim IAS Test Series (2019) – GS Test 10 (10.02.2019)


Economy 2 and Current Affairs Nov 2018 33
Source:
http://pib.nic.in/newsite/PrintRelease.a 66. Which of the following neighbours
spx?relid=181368 of India stood ahead of it in the
Global Hunger Index Report 2018?
65. Consider the following statements (1) Sri Lanka
about Bhoomi Rashi: (2) Myanmar
(1) It is a portal comprising of the (3) Nepal
entire revenue data of the (4) Bangladesh
country. Select the correct answer using the
(2) It has been integrated with code given below:
Public Financial Management (a) 1 and 3 only
Systems. (b) 2 and 4 only
(3) It will facilitate payment relate (c) 1, 2 and 3 only
to the compensation for land (d) 1, 2, 3 and 4
acquisition to all beneficiaries
directly through it. Answer: (d)
Which of the statements given Explanation:
above are correct? The Global Hunger Index was recently
(a) 1 and 2 only released in October 2018. It is a peer-
(b) 2 and 3 only reviewed publication released annually
(c) 1 and 3 only by Welthungerhilfe and Concern
(d) 1, 2 and 3 Worldwide. Overall, India has been
ranked at 103 out of 119 countries in the
Answer: (d) Index, with hunger levels in the country
Explanation: categorised as “serious”. Its ranking has
Bhoomi Rashi is a portal that has been dropped three places from last year.
developed in collaboration with National Many neighbouring countries like China
Informatics Centre, comprising the (25th), Sri Lanka (67th), Myanmar (68th),
entire revenue data of the country. The Nepal (72nd) and Bangladesh (86th)
entire process flow, from submission of stood ahead of India.
draft notification by the State
Government to its approval by the 67. Which among the following
Ministry is online. It has been integrated measures have been taken by
with Public Financial Management Government of India to support the
Systems. With this payment of renewable energy sector?
Compensation by the Ministry to the (1) Viability Gap Funding
beneficiaries will be just-in-time and (2) Maximum 50% FDI under
without any parking of funds. It will act automatic route
as one of the key functionalities to (3) Transparent bidding process
facilitate payment relate to the Select the correct answer using the
compensation for land acquisition to code given below:
all beneficiaries directly through it. (a) 1 only
Hence, Statements 1, 2 and 3 are (b) 2 only
correct. (c) 1 and 3 only
Source: (d) 1, 2 and 3
http://pib.nic.in/newsite/PrintRelease.a
spx?relid=181517 Answer: (c)

Prelim IAS Test Series (2019) – GS Test 10 (10.02.2019)


Economy 2 and Current Affairs Nov 2018 34
Explanation: development banks set up by advanced
The ambitious goal of generating 175 economies, AIIB is the first major
GW of renewable energy by 2022, and multilateral development bank where
initiatives on smart cities, electric principal contributors are the borrowing
vehicles, energy efficiency initiatives and members themselves.
others have now made India one of the
global leaders in climate action. By 69. With reference to Special
November 2018, a total capacity of Economic Zones (SEZs), consider
around 73.95GW has been installed in the following statements:
the country of which 37.84GW has been (1) It is a geographical region
added during the last four and half years. that has economic laws
Following measures have been different from country’s
implemented by Government of India to typical economic laws.
support the renewable energy sector: (2) The supplies to SEZs are
 Viability Gap Funding zero rated under IGST Act,
 Permitting FDI upto 100 percent 2007.
under automatic route (3) Recently, government has
 Fiscal Incentives such as capital constituted N. K Singh
subsidy, accelerated depreciation, Committee to review the
waiver of Inter State Transmission Policy regarding SEZs.
System (ISTS) charges and losses Which of the statements given
 Transparent bidding process i.e above are correct?
through e-reverse auction. (a) 1 and 2 only
(b) 2 and 3 only
68. Unlike most other multilateral (c) 1 and 3 only
development banks set up by (d) 1, 2 and 3
advanced economies, Asian
Infrastructure Investment Bank Answer: (a)
(AIIB) is the first major multilateral Explanation:
development bank where principal With a view to attract larger foreign
contributors are the borrowing investments in India, the Special
members themselves. Which one Economic Zones (SEZs) Policy was
of the following is the largest announced in April 2000.This policy
borrower from AIIB? intended to make SEZs an engine for
(a) Bangladesh economic growth supported by quality
(b) Myanmar infrastructure complemented by an
(c) Sri Lanka attractive fiscal package, both at the
(d) India Centre and the State level, with the
minimum possible regulations.
Answer: (d) Statement 1 is correct.
Explanation: Special Economic Zone (SEZ) is a
With the total project portfolios of US specifically delineated duty-free enclave
$4.4 Billion, India has been the largest and shall be deemed to be foreign
borrower of Asian Infrastructure territory for the purposes of trade
Investment Bank (AIIB) lending since the operations and duties and tariffs. In order
time the Bank started its operations. words, SEZ is a geographical region that
Unlike most other multilateral

Prelim IAS Test Series (2019) – GS Test 10 (10.02.2019)


Economy 2 and Current Affairs Nov 2018 35
has economic laws different from a nutrition and health
country's typical economic laws. education to mothers.
The main objectives of the SEZ Act are: (3) Under the scheme Rs. 6000
 generation of additional economic are provided to BPL women
activity as maternity benefits to
 promotion of exports of goods promote institutional
and services deliveries.
 promotion of investment from Which of the statements given
domestic and foreign sources above is/are correct?
 creation of employment (a) 1 only
opportunities (b) 1 and 2 only
 development of infrastructure (c) 1 and 3 only
facilities (d) 1, 2 and 3
Statement 2 is correct.
The units in SEZs are exempted from Answers: (b)
Central Sales Tax, Service Tax and State Explanation:
sales tax. These have now subsumed Statement 1 is correct.
into GST and supplies to SEZs are zero The Integrated Child Development
rated under IGST Act, 2017. Services (ICDS) Scheme (launched in
Statement 3 is incorrect. 1975) is one of the flagship programmes
Recently, Baba Kalyani committee of the Government of India and
constituted by the Ministry of Commerce represents one of the world’s largest and
& Industry to study the existing SEZ unique programmes for early childhood
policy of India submitted its report to the care and development. The beneficiaries
government. under the Scheme are children in the age
Source: group of 0-6 years, pregnant women and
http://sezindia.nic.in/cms/introduction. lactating mothers.
php Statement 2 is correct.
https://economictimes.indiatimes.com Objectives of the Scheme are:
/news/economy/policy/what-is-  to improve the nutritional and health
special-economic- status of children in the age-group 0-
zone/articleshow/1164460.cms 6 years;
 to lay the foundation for proper
70. Integrated Child Development psychological, physical and social
Services (ICDS) is one of the development of the child;
world’s largest and unique  to reduce the incidence of mortality,
programmes for early childhood morbidity, malnutrition and school
care and development. In the dropout;
context of ICDS, consider the  to achieve effective co-ordination of
following statements: policy and implementation amongst
(1) Apart from children, other the various departments to promote
beneficiaries under the child development; and
scheme include pregnant  to enhance the capability of the
women and lactating mother to look after the normal health
mothers. and nutritional needs of the child
(2) One of the objectives of the through proper nutrition and health
scheme is to provide proper education.

Prelim IAS Test Series (2019) – GS Test 10 (10.02.2019)


Economy 2 and Current Affairs Nov 2018 36
Statement 3 is incorrect. (3) PMBJP does not cover the
Under the scheme there is no component surgical consumables and
of cash transfer. The third statement is stents.
related to Pradhan Mantri Matru Which of the statements given
Vandana Yojana. above is/are correct?
Source: (a) 1 only
http://pib.nic.in/newsite/PrintRelease.asp (b) 1 and 2 only
x?relid=173550 (c) 2 and 3 only
(d) 1, 2 and 3
71. Mission SAMPARK which is seen
in news sometimes is related to Answers: (b)
which of the following? Explanation:
(a) Sex workers Pradhan Mantri Bhartiya Jan Aushadhi
(b) Primitive tribes Pariyojana (PMBJP) is a campaign
(c) HIV infected people launched by the Department of
(d) Old age people Pharmaceuticals, Government of India,
to provide quality medicines at affordable
Answers: (c) prices to the masses through special
Explanation: kendra known as Pradhan Mantri
Mission SAMPARK is a scheme Bhartiya Jan Aushadhi Kendra. PMBJPK
implemented by The National AIDS has been set up to provide generic
Control Organization (NACO) and the drugs, which are available at lesser
Union Ministry of Health and Family prices but are equivalent in quality and
Welfare. It aims to eradicate HIV/AIDS by efficacy as expensive branded drugs.
2030. It is focused on reaching out to Statement 1 is correct.
track those who are left to follow up and BPPI (Bureau of Pharma PSUs of India),
are to be taken under Antiretroviral under the administrative control of the
therapy (ART) services. Department of Pharmaceuticals, Ministry
Source: of Chemicals & Fertilizers, Government
http://pib.nic.in/newsite/PrintRelease.a of India will be the implementation
spx?relid=174032 agency for the PMBJP.
Statement 2 is correct. Pradhan Mantri
72. The Pradhan Mantri Bhartiya Bhartiya Jan Aushadhi Pariyojana will be
Janaushadhi Pariyojana (PMBJP) implemented in all the districts of the
is launched with an objective of country.
making available quality generic Statement 3 is incorrect.
medicines at affordable prices to The scheme covers wide variety of
all, especially the poor. In the product including 700 Drugs & 154
context of PMBJP, consider the Surgical Items and stents as well.
following statements: Source:
(1) Bureau of Pharma PSUs of http://janaushadhi.gov.in/pmjy.aspx
India (BPPI) is the
implementation agency for 73. The Basel Convention, often seen
the PMBJP. in news, is related to which of the
(2) PMBJP is implemented in all following?
districts of the country. (a) Rules for illegal migrants in
Europe.

Prelim IAS Test Series (2019) – GS Test 10 (10.02.2019)


Economy 2 and Current Affairs Nov 2018 37
(b) Banking sector regulations.
(c) Non-proliferation of nuclear Answer: (c)
weapons. Explanation:
(d) Transboundary Movements Statement 1 is correct.
of Hazardous Wastes and Arsenic accumulation in rice grains is
Their Disposal. one of the serious agricultural issues in
India. To address this, researchers at
Answer: (d) Lucknow- based CSIR-National
Explanation: The Basel Convention on Botanical Research Institute have
the Control of Transboundary developed transgenic rice by inserting a
Movements of Hazardous Wastes and novel fungal gene, which results in
Their Disposal, usually known as the reduced arsenic accumulation in rice
Basel Convention, is an international grain.
treaty that was designed to reduce the Statement 2 is correct.
movements of hazardous waste between Transgenic rice promises to generate
nations, and specifically to prevent high yields even under conditions of high
transfer of hazardous waste from salinity, high temperature and drought.
developed to less developed countries
(LDCs). It does not, however, address 75. India has the largest number of
the movement of radioactive waste. The malnourished children in the world.
Convention is also intended to minimize In this context, consider the
the amount and toxicity of wastes following statements:
generated, to ensure their (1) Odisha has a better
environmentally sound management as nutritional status compared
closely as possible to the source of with Gujarat.
generation, and to assist LDCs in (2) Chhattisgarh has improved
environmentally sound management of malnutrition level compared
the hazardous and other wastes they to Maharashtra.
generate. Which of the statements given
Source: above is/are correct?
http://www.basel.int/TheConvention/O (a) 1 only
verview/tabid/1271/Default.aspx (b) 2 only
(c) Both 1 and 2
74. What are the benefits of 'transgenic (d) Neither 1 nor 2
rice', developed in India?
(1) It can help to reduce arsenic Answer: (c)
accumulation in rice grains. Explanation:
(2) It can generate high yields Both statements 1 and 2 are correct:
even under the conditions of The low income and Empowered-Action-
high salinity, high Group (EAG) States face major
temperature and drought. challenges to improve malnutrition, but,
Select the correct answer using the two EAG States, Chhattisgarh and
code given below: Odisha, have performed better on this
(a) 1 only front compared to Gujarat and
(b) 2 only Maharashtra where per capita income is
(c) Both 1 and 2 almost double. Odisha, which is a low-
(d) Neither 1 nor 2 income State, has a better network of

Prelim IAS Test Series (2019) – GS Test 10 (10.02.2019)


Economy 2 and Current Affairs Nov 2018 38
Integrated Child Development Services educational institutes of nation, as they
(ICDS), public health facility/workforce are the primary target of the
per lakh population and educational manufacturing companies to pick their
attainment among women, which have potential employees. With the
translated into a better nutritional status introduction of the UAY, the students
when compared with Gujarat. from those educational institutes able to
Additional Information: gain practical knowledge of those
The launch of the National Nutrition modern technologies which ease the
Mission as a strategy to fight maternal process of manufacturing units and
and child malnutrition is a welcome step industrial plants to boost its growth.’
towards achieving the targets of During the initial stage the UAY will be
underweight and stunted children under implemented only among the premier
five years from 35.7% to 20.7% and from educational units across the nations such
38.4% to 25% respectively by 2022. as IITs and so. The implementation will
be carried along with the partnership of
76. Consider the following statements the respective industries.
with reference to the Uchchatar
Avishkar Yojana (UAY): 77. With reference to the 'Regional
(1) The scheme is launched by Integrated Multi-Hazard Early
the Ministry of Science and Warning System (RIMES)', which
Technology. was in the news recently, consider
(2) The main objective of the the following statements:
scheme is to bridge the gap (1) It is an inter-governmental
between academics and body of African and Asian
practical working field. nations registered under the
(3) Presently, the scheme is United Nations.
being implemented only in (2) It was established in 2005, in
premier educational the aftermath of the 2004
institutions of the country. Indian Ocean tsunami.
Which of the statements given (3) It operates from its regional
above is/are correct? early warning centre located
(a) 1 only at the campus of the Indian
(b) 1 and 2 only Institute of Technology-
(c) 2 and 3 only Madras, India.
(d) 1, 2 and 3 Which of the statements given
above is/are correct?
Answer: (c) (a) 1 only
Explanation: (b) 1 and 2 only
Statement 1 is incorrect. (c) 2 and 3 only
Uchchatar Avishkar Yojana (UAY) is (d) 1, 2 and 3
launched by Ministry of Human
Resources Development and handled by Answer: (a)
Department of Higher Education. Explanation:
Statements (2) and (3) are correct. Statement 1 is correct.
The main motive of the scheme is to The Regional Integrated Multi-Hazard
introduce the latest and modern Early Warning System for Africa and
technology to the students of the premier Asia (RIMES) is an international and

Prelim IAS Test Series (2019) – GS Test 10 (10.02.2019)


Economy 2 and Current Affairs Nov 2018 39
intergovernmental institution, owned and them. It is based on the principles of
managed by its Member States, for the Open Regionalism for strengthening
generation and application of early Economic Cooperation particularly on
warning information. Trade Facilitation and Investment,
RIMES evolved from the efforts of Promotion as well as Social
countries in Africa and Asia, in the Development of the region. The
aftermath of the 2004 Indian Ocean Coordinating Secretariat of IORA is
tsunami, to establish a regional early located at Ebene, Mauritius.
warning system within a multi-hazard In the recent Indian Ocean Rim
framework for the generation and Association (IORA) meeting, the
communication of early warning Maldives was accepted as the 22nd
information, and capacity building for member of the IORA, while Myanmar’s
preparedness and response to trans- membership application was not
boundary hazards. approved due to lack of consensus.
Statement 2 is incorrect. Source: Vajiram and Ravi Current
RIMES was established on 30 April Affairs booklet - November 2018, Page
2009, and was registered with the United 57
Nations on 1 July 2009.
Statement 3 is incorrect. 79. With reference to the 'Indian Health
RIMES operate from its regional early Fund (IHF)', sometimes seen in the
warning center located at the campus of news, consider the following
the Asian Institute of Technology in statements:
Pathumthani, Thailand. (1) IHF was launched by the
Ministry of Health and Family
78. Which of the following is the most Welfare in collaboration with
recent nation to be inducted in the NITI Aayog.
Indian Ocean Rim Association (2) The Fund mainly supports
(IORA)? individuals and organizations
(a) Myanmar for sustainable and scalable
(b) Maldives solutions in addressing
(c) Eritrea Japanese encephalitis and
(d) South Africa dengue.
Which of the statements given
Answer: (b) above is/are correct?
Explanation: (a) 1 only
The Indian Ocean Rim Association (b) 2 only
(IORA), formerly known as the Indian (c) Both 1 and 2
Ocean Rim Initiative and Indian Ocean (d) Neither 1 nor 2
Rim Association for Regional
Cooperation (IOR-ARC), is an Answer: (d)
international organisation consisting of Explanation:
coastal states bordering the Indian Statement 1 is incorrect.
Ocean. The IORA is a regional forum, Indian Health Fund (IHF) was launched
tripartite in nature, bringing together in 2016 by Tata Trusts in collaboration
representatives of Government, Business with the Global Fund, to drive innovations
and Academia, for promoting co- towards key infectious disease
operation and closer interaction among challenges in India.

Prelim IAS Test Series (2019) – GS Test 10 (10.02.2019)


Economy 2 and Current Affairs Nov 2018 40
Statement 2 is incorrect. technologies. The activities of AMF TCP
It supports individuals and organisations are deployment and dissemination of
with already germinated innovative Advanced Motor Fuels. It looks upon the
strategies, services, products, such that transport fuel issues in a systemic way
they become sustainable and scalable taking into account the production,
solutions in addressing TB and malaria. distribution and end-use related aspects.
The initiative is a long-term exercise Statement 2 is incorrect.
aligned with country’s goal of eliminating Ministry of Petroleum & Natural Gas has
TB by 2025 and malaria by 2030. It will joined AMF TCP as its 16th member in
promote innovative solutions such that 2018. The other member Countries of
they are widely accessible and are AMF TCP are USA, China, Japan,
affordable. Canada, Chile, Israel, Germany, Austria,
Sweden, Finland, Denmark, Spain,
80. Regarding the ‘Advanced Motor Republic of Korea, Switzerland and
Fuels Technology Collaboration Thailand.
Programme (AMF TCP)’, which Statement 3 is correct.
was in the news recently, consider AMF TCP also provides an opportunity
the following statements: for fuel analysis, identifying new/
(1) AMF TCP works under the alternate fuels for deployment in
framework of Fuel cell and transport sector and allied R&D activities
Hydrogen Energy for reduction in emissions in fuel
Association. intensive sectors.
(2) Ministry of Road Transport
and Highways has joined 81. With reference to the ‘National
AMF TCP as its 25th Register of Citizens (NRC)’,
member. sometimes seen in the news,
(3) The programme is an consider the following statements:
international platform for co- (1) NRC is the only procedure to
operation among countries to establish citizenship and
promote cleaner and more identify foreigners in Assam.
energy efficient fuels and (2) In Assam the NRC was first
vehicle technologies. updated in 1971 and since
Which of the statements given then it has not been updated.
above is/are correct? (3) The Citizenship Act, 1955
(a) 3 only provides for compulsory
(b) 2 and 3 only registration of every citizen of
(c) 1 and 3 only India.
(d) 1, 2 and 3 Which of the statements given
above is/are correct?
Answer: (a) (a) 1 and 2
Explanation: (b) 3 only
Statement 1 is incorrect. (c) 1, 2 and 3
AMF TCP is an international platform (d) None of the above
under the framework of International
Energy Agency (IEA) for co-operation Answer: (b)
among countries to promote cleaner and Explanation:
more energy efficient fuels & vehicle

Prelim IAS Test Series (2019) – GS Test 10 (10.02.2019)


Economy 2 and Current Affairs Nov 2018 41
After a militant movement to protect progress in Gorakhpur Haryana Anu
Assam against Bangladesh infiltrators, Vidyut Pariyojana (GHAVP) Units -1&2
the Assam Accord was signed in 1985 (2X700 MW) at an advanced stage. The
between the Government of India and All first phase of the project comprising of
Assam Students Union (AASU), which two units GHAVP-1 & 2 (2X700 MW) is
led to an amendment in the Citizenship expected to be completed in 2025.
Act, 1955, to incorporate Section 6A Chutka in Madhya Pradesh. And
specifically pertaining to Assam. Rawatbhata is in Rajasthan.
Under this specific section, 100
Foreigners Tribunals have been 83. Which of the following international
established in Assam to identify airport in India received the
foreigners. Section 6A says all Indian ‘Champions of Earth’ award
origin persons, including from recently?
Bangladesh who entered Assam before (a) Veer Savarkar International
January 1966, are deemed citizens. Airport, Andaman and
Those who came between January 1, Nicobar Islands
1966, and March 25, 1971, can also get (b) Kempegowda International
citizenship after registering themselves Airport, Karnataka
and living in India for 10 years. (c) Cochin International Airport,
Everyone who entered after March 25, Kerala
1971, is to be identified as foreigner by (d) Sardar Vallabhbhai Patel
the Tribunals and deported. In Assam the International Airport, Gujarat
updating of NRC could not be carried on
since 1951 due to various political Answer: (c)
tensions such as Assam Movement of Explanation:
1980s, the language movement etc. Recently the Prime Minister of India and
the Cochin airport were awarded with the
82. Consider the following pairs: 'Champions of the Earth' award. The
Atomic State ‘Champions of the Earth award’ was
Power launched in 2005. “Champions of the
Plant in India Earth”, the UN’s highest environmental
1. Gorakhpur Uttar honour, celebrates outstanding figures
Pradesh from the public and private sectors and
2. Rawatbhata Rajasthan from civil society whose actions have had
3. Chutka Madhya a transformative positive impact on the
Pradesh environment.
Which of the above pairs is/are
correctly matched? 84. The term ‘She-Box’, is seen in the
(a) 1 only news sometimes. Consider the
(b) 2 and 3 only following statements in this context.
(c) 1 and 3 only (1) Under the She-Box initiative,
(d) 1, 2 and 3 sanitary pad vending
machines are installed in
Answer: (b) schools and colleges for
Explanation: promoting girls and woman's
Gorakhpur atomic power plant is in health.
Haryana, presently excavation is in

Prelim IAS Test Series (2019) – GS Test 10 (10.02.2019)


Economy 2 and Current Affairs Nov 2018 42
(2) The Initiative was launched Combination products, also known as
by the World Health fixed dose drug combinations (FDCs),
Organization in partnership are combinations of two or more active
with the Government of India. drugs in a single dosage form. FDCs’
Which of the statements given popularity in India is due to advantages
above is/are correct? such as increased efficacy, better
(a) 1 only compliance, reduced cost and simpler
(b) 2 only logistics of distribution. FDCs have
(c) Both 1 and 2 shown to be particularly useful in the
(d) Neither 1 nor 2 treatment of infectious diseases like HIV,
malaria and tuberculosis, where giving
Answer: (d) multiple antimicrobial agents is the norm.
Explanation: FDCs are also useful for chronic
Statement 1 and (2) are incorrect. conditions especially, when multiple
It is an online complaint management disorders co-exist. According to a study,
system for registering complaints related of the 110 anti-TB (tuberculosis) Fixed
to sexual harassment at workplace. It Dose Combinations (FDCs) available in
was launched by the Ministry of Women India, only 32 (less than 30%) have been
and Child Development. The complaint approved by the Central Drugs Standard
management system has been Control Organisation (CDSCO), the
developed to ensure the effective country’s drug regulator.
implementation of Sexual Harassment of
Women at Workplace (Prevention, 86. With reference to 'Kambala', an
Prohibition and Redressal) Act (the SH annual buffalo race, which of the
Act), 2013. following statements is/are correct?
(1) It is a traditional festival of
85. Which of the following is/are the the farming community of
advantages of Fixed Dose Karnataka.
Combinations (FDCs) which are (2) The event is mainly
the combinations of two or more organized during rainy
active drugs in a single dosage season in the months of July
form? and August.
(1) Better compliance. Select the correct answer using the
(2) Useful in the treatment of code given below:
infectious diseases. (a) 1 only
(3) Useful for chronic conditions (b) 2 only
especially when multiple (c) Both 1 and 2
disorders co-exist. (d) Neither 1 nor 2
Select the correct answer using the
code given below: Answer: (a)
(a) 1 only Explanation:
(b) 1 and 2 only Statement 1 is correct.
(c) 2 and 3 only Kambala is an annual buffalo race which
(d) 1, 2 and 3 is a tradition in the coastal region of
Karnataka (Dakshina Kannada and
Answer: (d) Udupi districts and bordering area of
Explanation:

Prelim IAS Test Series (2019) – GS Test 10 (10.02.2019)


Economy 2 and Current Affairs Nov 2018 43
Kerala). This area is called Tulunadu littoral zone is the part of a sea, lake or
(Land of Tulu Language). river that is close to the shore.
Statement 2 is incorrect. Statement 2 is incorrect.
This annual event starts in November Launched in 2008, biennial meetings
and lasts till March every year. According among the littoral states of the Indian
to one belief, Kambala is a festival that Ocean region have been held ever since.
originated in the farming community of The IONS act as a security construct for
Karnataka around 800 years back. The the Indian Ocean region and apart from
festival is dedicated to Kadri's Lord its series of symposiums, it conducts
Manjunatha, an incarnation of Lord numerous other activities like workshops,
Shiva. It is believed to be celebrated to essay competitions and lectures to
please the Gods for a good harvest. It is promote its objective.
a form of recreational sport for the
farming community and also another 88. Which one of the following is the
belief is that it is considered as the Sport best description of ‘Shakti’ that was
of the royal (landlord) Family. in the news recently?
(a) India’s first indigenous
87. In the context of the 'Indian Ocean microprocessor designed by
Naval Symposium (IONS)', which IIT-Madras.
of the following statements is/are (b) Indigenously developed
correct? medium-range mobile
(1) Pakistan is the only littoral surface-to-air missile defense
state of the Indian Ocean system developed by the
region which is not a member DRDO.
of the IONS. (c) India’s first indigenous
(2) The annual meetings of the cryogenic engine developed
IONS have been held by ISRO
regularly in the month of (d) Indigenously developed low-
October since its launched in cost smart home system
2015. designed by IIT-Delhi.
Select the correct answer using the
code given below: Answer: (a)
(a) 1 only Explanation:
(b) 2 only Indian Institute of Technology Madras
(c) Both 1 and 2 (IIT Madras) researchers have designed
(d) Neither 1 nor 2 India’s first indigenous microprocessor
called ‘Shakti’. It is aimed at developing
Answer: (d) industrial-grade microprocessors and
Explanation: other components of the microprocessor
Statement 1 is incorrect. ecosystem. It was designed, developed
The Indian Ocean Naval Symposium is a and booted by IIT Madras with microchip
voluntary initiative that seeks to increase fabricated in ISRO’s Semi-Conductor
maritime co-operation among navies of Laboratory at Chandigarh. It has been
the littoral states of the Indian Ocean developed under project partly funded by
Region by providing an open and Ministry of Electronics and Information
inclusive forum for discussion of Technology (MeitY), as part of two-
regionally relevant maritime issues. The

Prelim IAS Test Series (2019) – GS Test 10 (10.02.2019)


Economy 2 and Current Affairs Nov 2018 44
decade-old efforts to develop indigenous Institution's National Museum of Natural
microprocessors. History). In February 2017, at major
conference on genomics and biodiversity
89. Regarding the ‘Earth Bio-Genome organized by the Smithsonian Institution
Project’, which was in the news and BGI in Washington, D.C. was
recently, consider the following supported project's 10-year plan and
statements: organizational structure.
(1) It proposes a detailed
genome-sequence draft of 90. With reference to the ‘New
every organisms with a Horizons Mission’, sometimes seen
defined nucleus and to which in the news, consider the following
belong all plants and statements:
animals. (1) It is the first spacecraft which
(2) It is an international, successfully fly by the planet
collaborative research Mercury.
program initiated by the (2) This mission is a part of
World Health Organizations. NASA’s Viking program.
Which of the statements given Which of the statements given
above is/are correct? above is/are correct?
(a) 1 only (a) 1 only
(b) 2 only (b) 2 only
(c) Both 1 and 2 (c) Both 1 and 2
(d) Neither 1 nor 2 (d) Neither 1 nor 2

Answer: (a) Answer: (d)


Explanation: Statement 1 and (2) are incorrect.
Statement 1 is correct. New Horizons is an interplanetary space
The Earth Bio-Genome Project (EBP) probe that was launched as a part of
plans to record the genomes - the DNA NASA’s New Frontiers program. It was
blueprint of life - of 1.5 million species of launched in 2006, became the first
animal, plant, protozoa and fungi within a mission to visit Pluto in 2015. Travelling
decade. So far, 19 research institutions farther into the Kuiper Belt, the nuclear-
around the world have signed up to take powered space probe has come within
part in the EBP and more plan to join. 3,500 km of Ultima Thule. It is the fifth
They expect to read the full DNA artificial object to achieve the escape
sequence of all the world’s eukaryotic velocity needed to leave the Solar
species - organisms whose cells have a System. New Horizon’s core science
nucleus enclosed by membranes. These mission is to map the surfaces of Pluto
are animals, plants, fungi and protozoa, and Charon, to study Pluto’s atmosphere
which encompass all of life except simple and to take temperature readings.
microbes (bacteria and archaea).
Statement 2 is incorrect. 91. Which of the statements is/are
The initiative was inspired by Human correct regarding the ‘National
Genome Project, and emerged during Accreditation Board for Certification
November 2015 meeting between Harris Bodies (NABCB)’, sometimes seen
Lewin (UCD), Gene E. Robinson (IGB) in the news?
and W. John Kress (Smithsonian

Prelim IAS Test Series (2019) – GS Test 10 (10.02.2019)


Economy 2 and Current Affairs Nov 2018 45
(1) NABCB is a constituent of (1) The framework is an
Quality Council of India agreement between the UN
(QCI). Security Council and the
(2) NABCB is the third intelligence agencies of all
accreditation body in the the members of the United
Asia-Pacific Region to Nations.
become internationally (2) The Security Council of the
equivalent in the region. United Nations will oversee
Select the correct answer using the and monitor the
code given below: implementation of the
(a) 1 only framework.
(b) 2 only Which of the statements given
(c) Both 1 and 2 above is/are correct?
(d) Neither 1 nor 2 (a) 1 only
(b) 2 only
Answer: (c) (c) Both 1 and 2
Explanation: (d) Neither 1 nor 2
Statement 1 is correct.
The National Accreditation Board for Answer: (d)
Certification Bodies (NABCB), India’s Explanation:
national accreditation body, has secured The United Nations has launched a new
equivalence for its accreditation framework titled ‘UN Global Counter-
programme for Occupational Health and Terrorism Coordination Compact’ to
Safety Management Systems (OHSMS). combat international terrorism and
NABCB can facilitate export of Indian coordinate efforts across the peace and
goods in the global market by attesting security, humanitarian, human rights and
that these are certified as per sustainable development sectors.
international standards by competent Statement 1 is incorrect.
certifying bodies. It is a constituent Board The framework is an agreement between
of the Quality Council of India, is the UN chief, 36 organisational entities,
responsible for accreditation of the International Criminal Police
certification/inspection bodies as per Organisation (INTERPOL) and the World
applicable international standards under Customs Organisation to better serve the
an international system of equivalence. needs of member states when it comes
Statement 2 is correct. to tackling the scourge of international
It is the third accreditation body in the terrorism.
Asia-Pacific region (other two being the Statement 2 is incorrect.
accreditation bodies of Hong Kong and The Coordination Committee of the
Mexico). United Nations will oversee the
implementation of the framework and
92. The United Nations has recently monitor its implementation. The
launched a new framework titled committee will be chaired by UN Under-
‘UN Global Counter-Terrorism Secretary-General for counter-terrorism.
Coordination Compact’, in this
context, consider the following 93. Regarding the 'Organisation for the
statements: Prohibition of Chemical Weapons
(OPCW), sometimes seen in the

Prelim IAS Test Series (2019) – GS Test 10 (10.02.2019)


Economy 2 and Current Affairs Nov 2018 46
news, consider the following India has signed the agreement in 1993
statements: and ratified it in 1996.
(1) The OPCW has the powers Statement 3 is incorrect.
to monitor chemical industry The OPCW was awarded the 2013 Nobel
to prevent chemical weapons Peace Prize for its work in eliminating
from re-emerging. chemical weapons.
(2) India has signed the OPCW
agreement but is yet to ratify 94. Regional coalition known as the
it. ‘Quad’, is a quadrilateral formation,
(3) The OPCW was awarded the which of the following countries is
2015 Nobel Peace Prize for not part of it?
its work in Syria. (a) Japan
Which of the statements given (b) India
above is/are correct? (c) Australia
(a) 1 only (d) Singapore
(b) 1 and 2 only
(c) 2 and 3 only Answer: (d)
(d) 1, 2 and 3 Explanation:
Regional coalition known as the ‘Quad’,
Answer: (a) the quadrilateral formation includes
Explanation: Japan, India, United States and
The Organisation for the Prohibition of Australia. All four nations find a common
Chemical Weapons (OPCW) is a Hague- ground of being the democratic nations
based intergovernmental body that works and common interests of unhindered
for the elimination of chemical weapons. maritime trade and security. The idea
It was formed after the Chemical was first mooted by Japanese Prime
Weapons Convention - an arms control Minister Shinzo Abe in 2007.
treaty that bans the production,
stockpiling and use of chemical weapons 95. With reference to the 'Trans fatty
- entered into force in 1997. The OPCW acids (TFAs)', sometimes seen in
is the implementing body of the the news, in this regard, which of
Convention. All its 192-member states the following statements is/are
are required to destroy their existing correct?
stockpiles of chemical weapons and stop (1) Partially hydrogenated
large-scale production. These actions are Vegetable Oils (PHVOs) are
subject to verification by the OPCW. the major source of trans-
According to the OPCW, over 96% of the fatty acids in India.
world’s declared chemical weapons (2) Food Safety and Standards
stockpiles have been destroyed. Authority of India (FSSAI)
Statement 1 is correct. has proposed to limit TFA
The OPCW has the powers to inspect limit in foods to 1% and
chemical production facilities. Israel has eliminate trans fats from
signed the agreement but is yet to ratify foods by 2025.
it. The countries that have not signed it Select the correct answer using the
are Egypt, South Sudan and North code given below
Korea. (a) 1 only
Statement 2 is incorrect. (b) 2 only

Prelim IAS Test Series (2019) – GS Test 10 (10.02.2019)


Economy 2 and Current Affairs Nov 2018 47
(c) Both 1 and 2 Select the correct answer using the
(d) Neither 1 nor 2 code given below:
(a) 1 only
Answer: (a) (b) 2 only
Explanation: (c) Both 1 and 2
Partially hydrogenated Vegetable Oils (d) Neither 1 nor 2
(PHVOs) are the major source of trans-
fatty acids in India. Trans fatty acids Answer: (b)
(TFAs) or Trans fats are the most Explanation:
harmful type of fats which can have The geo-engineering technique such as
much more adverse effects on our body stratospheric aerosol injection (SAI)
than any other dietary constituent. These could limit rising temperatures that are
fats are largely produced artificially but a causing climate change.
small amount also occurs naturally. TFAs Statement 1 is incorrect.
pose a higher risk of heart disease than Under SAI delivery of precursor sulphide
saturated fats. While saturated fats raise gases such as sulfuric acid, hydrogen
total cholesterol levels, TFAs not only sulphide (H2S) or sulphur dioxide (SO2)
raise total cholesterol levels but also are sprayed by artillery, aircraft and
reduce the good cholesterol (HDL), balloons.
which helps to protect us against heart Statement 2 is correct.
disease. Trans fats consumption This proposed method could counter
increases the risk of developing heart most climatic changes including
disease and stroke. WHO recommends temperature rise, take effect rapidly,
that trans-fat intake be limited to less have very low direct implementation
than 1% of total energy intake and has costs, and be reversible in its direct
called for the total elimination of TFAs in climatic effects.
global food supply by 2023. FSSAI has
proposed to limit TFA limit in foods to 2% 97. With reference to the 12th 'Asia-
and eliminate trans fats from foods by Europe Meeting (ASEM)' summit
2022. held in Belgium, which of the
following statements is/are correct?
96. Which of the following statements (1) It is a formal process of
is/are correct regarding the dialogue between the
Stratospheric Aerosol Injection member nations to address
(SAI), sometimes seen in the mainly the security and
news? environment related issues.
(1) SAI technique is a proposal (2) The ASEM Summit is an
to spray large quantities of annual meeting between the
carbon dioxide into the defence and environment
stratosphere to act as a ministers of the member
reflective barrier against countries.
incoming sunlight. (3) It was officially established
(2) The main objective of the SAI on 1 March 1996 at the first
is to limit the rising summit in Bangkok, Thailand.
temperatures that are Select the correct answer using the
causing climate change. code given below:
(a) 2 only

Prelim IAS Test Series (2019) – GS Test 10 (10.02.2019)


Economy 2 and Current Affairs Nov 2018 48
(b) 3 only differently-able people
(c) 1 and 2 only around the world.
(d) 1 and 3 only Which of the statements given
above is/are correct?
Answer: (b) (a) 1 only
Explanation: (b) 2 only
The Asia-Europe Meeting (ASEM) is an (c) Both 1 and 2
informal process of dialogue and (d) Neither 1 nor 2
cooperation bringing together the 28
European Union member states, 2 other Answer: (c)
European countries, and the European Explanation:
Union with 21 Asian countries and the Statement 1 is correct.
ASEAN Secretariat. Union Ministry of External Affairs (MEA)
Statement 1 is incorrect. has launched India for Humanity initiative
The ASEM Summit is a biennial meeting to commemorate the 150th birth
between the Heads of State and anniversary of Mahatma Gandhi and
Government, the President of the honour of his service to humanity. It will
European Council, the President of the feature year-long series of artificial limb
European Commission, and the fitment camps in a number of countries
Secretary-General of the Association of spanning globe. For this initiative, MEA
Southeast Asian Nations (ASEAN). has collaborated with renowned
Statement 2 is incorrect. charitable organisation Bhagwan
The ASEM dialogue addresses political, Mahaveer Viklang Sahayata Samiti
economic and cultural issues, with the (BMVSS).
objective of strengthening the Statement 2 is correct.
relationship between the two regions, in a It will provide physical, economic and
spirit of mutual respect and equal social rehabilitation of differently-able
partnership. around world by helping them regain
Statement 3 is correct. their mobility and dignity to become self-
It was officially established on 1 March respecting and productive members of
1996 at the first summit in Bangkok, society. It focuses on Mahatma Gandhi’s
Thailand. philosophy of compassion, caring and
service to humanity.
98. Recently the Government of India
has launched the ‘India for 99. With reference to ‘the Conference
Humanity initiative’, in this context, on Disarmament (CD)’, sometimes
consider the following statements: seen in the news, consider the
(1) Union Ministry of External following statements:
Affairs (MEA) has launched (1) It is a multilateral
this initiative to disarmament forum
commemorate the 150th birth established by the United
anniversary of Mahatma Nations.
Gandhi. (2) India, Pakistan, Israel and
(2) Under this Initiative the main North Korea are not the
objective is to provide member of the conference.
physical, economic and
social rehabilitation of

Prelim IAS Test Series (2019) – GS Test 10 (10.02.2019)


Economy 2 and Current Affairs Nov 2018 49
(3) The conference meets (b) 2 only
annually in three separate (c) Both 1 and 2
sessions in Geneva. (d) Neither 1 nor 2
Which of the statements given
above is/are correct? Answer: (b)
(a) 2 only Explanation:
(b) 3 only Statement 1 is incorrect.
(c) 1 and 2 only NASA's Chandra X-ray Observatory, a
(d) 1 and 3 only space based x-ray observatory, was
launched and deployed by Space Shuttle
Answer: (b) Columbia on July 23, 1999. It is the most
Explanation: sophisticated X-ray observatory built to
The Conference on Disarmament (CD) is date.
a multilateral disarmament forum Statement 2 is correct.
established by the international Chandra is designed to observe X-rays
community to negotiate arms control and from high-energy regions of the universe,
disarmament agreements based at the such as the remnants of exploded stars.
Palais des Nations in Geneva. The
Conference meets annually in three
separate sessions in Geneva. The
Conference was first established in 1979
as the Committee on Disarmament as
the single multilateral disarmament
negotiating forum of the international
community. It was renamed the
Conference on Disarmament in 1984.
The Conference is formally independent
from the United Nations. The conference
has 65 members represent all areas of
the world, including all known nuclear-
weapon states including India, Pakistan,
Israel and North Korea.

100. Regarding the Chandra X-ray


Observatory (CXO), which of the
following statements is/are correct?
(1) It is a space-based satellite
observatory launched by
ISRO in 2009.
(2) It has been specially
designed to detect X-ray
emission from very high
energy regions of the
universe.
Select the correct answer using the
code given below:
(a) 1 only

Prelim IAS Test Series (2019) – GS Test 10 (10.02.2019)


Economy 2 and Current Affairs Nov 2018 50

S-ar putea să vă placă și